cardiovascular

¡Supera tus tareas y exámenes ahora con Quizwiz!

The nurse is assisting with an education conference for graduate nurses about infant CPR. Which of the following statements are appropriate to include in the teaching? Select all that apply. 1. "A single rescuer responding to an unwitnessed infant arrest should perform 2 minutes of CPR before retrieving a defibrillator." 2. "Depth of chest compressions for infants should be half the depth of the anterior-posterior chest diameter." 3. "Rescuers should place the heel of one hand on the lower sternum when delivering chest compressions to infants." 4. "The ratio of chest compressions to breaths during CPR by a single rescuer is 15:2 for infants." 5. "You should assess the infant's brachial pulse for no longer than 10 seconds."

1. "A single rescuer responding to an unwitnessed infant arrest should perform 2 minutes of CPR before retrieving a defibrillator." 5. "You should assess the infant's brachial pulse for no longer than 10 seconds." Educational objective:The American Heart Association provides guidelines for basic life support of infants, including initial client evaluation (eg, assess brachial pulse) and retrieval of automatic external defibrillator (ie, after 2 min of CPR during an unwitnessed collapse with a single rescuer).

The nurse has just completed discharge teaching for a client recently diagnosed with hypertension. Which of the following statements by the client indicate understanding of the Dietary Approaches to Stop Hypertension (DASH) diet? Select all that apply. 1. "I need to eat less red meat and more fresh vegetables." 2. "I'll limit drinking soda to only one at a time as an occasional treat." 3. "I'm going to replace potato chips with fruit during meals and snacking." 4. "I'm really going to miss drinking as much milk as I normally do." 5. "Taking the salt shaker off the table should be enough to reduce my sodium intake

1. "I need to eat less red meat and more fresh vegetables." 2. "I'll limit drinking soda to only one at a time as an occasional treat." 3. "I'm going to replace potato chips with fruit during meals and snacking." The Dietary Approaches to Stop Hypertension (DASH) diet is often suggested to clients with hypertension due to its ability to reduce blood pressure. The diet focuses on elimination or reduction of foods and beverages high in sodium, sugar, cholesterol, and trans or saturated fats, which all contribute to increased blood pressure. The DASH diet focuses on: Including fresh fruits and vegetables, and whole grains in the daily diet Choosing fat-free or low-fat dairy products Choosing meats lower in cholesterol (eg, fish, poultry) and alternate protein sources (eg, legumes) instead of red meats (Option 1) Limiting intake of sweets, foods high in sodium (eg, potato chips, frozen meals, canned foods), and sugary beverages to the occasional treat (Options 2 and 3) Educational objective:The Dietary Approaches to Stop Hypertension (DASH) diet is often recommended to reduce blood pressure in clients with hypertension. The client is taught to limit intake of sugar, sodium, cholesterol, and trans or saturated fats, and instead choose healthier options (eg, fresh fruit and vegetables, low-fat dairy products).

A nurse cares for a client after cardiac catheterization. During assessment of the groin site, the nurse notices that the dressing is saturated with blood and a small trickle leaks down the client's leg. What should the nurse do first? 1. Apply direct manual pressure at and above the skin puncture site 2. Call the health care provider to report active bleeding 3. Check the peripheral pulse distal to the catheterization site 4. Place a new pressure dressing over the catheterization site

1. Apply direct manual pressure at and above the skin puncture site xplanation Explanation: Cardiac catheterization helps assess and diagnose coronary artery disease (eg, coronary artery patency, atherosclerosis). A catheter is advanced to the heart through a vein (eg, femoral, antecubital) for right-sided heart catheterization or an artery (eg, brachial, femoral) for left-sided heart catheterization. After the procedure, a pressure dressing is applied and the client placed supine with the affected extremity flat for 2-6 hours to promote complete hemostasis. The nurse monitors vital signs, extremity integrity (eg, pulses, sensation, capillary refill), and dressings for indications of bleeding according to institution policy. If bleeding occurs, the nurse applies direct manual pressure to the vessel puncture site (ie, about 2.5 cm [1"] above the skin puncture site) to achieve hemostasis and keep the client hemodynamically stable (Option 1). The health care provider (HCP) should be notified, as the client may require further surgical intervention. Educational objective:If bleeding occurs after cardiac catheterization, the nurse first applies direct manual pressure to control the bleeding.

The nurse is reviewing discharge instructions on home management for a client with peripheral arterial disease. Which statements indicate a correct understanding of the instructions? Select all that apply. 1. "I will apply moisturizing lotion on my legs every day." 2. "I will elevate my legs at night when I am sleeping." 3. "I will keep my legs below heart level when sitting." 4. "I will start walking outside with my neighbor." 5. "I will use a heating pad to promote circulation."

1. "I will apply moisturizing lotion on my legs every day." 3. "I will keep my legs below heart level when sitting." 4. "I will start walking outside with my neighbor." Peripheral arterial disease (PAD) is a chronic, atherosclerotic disease caused by buildup of plaque within the arteries. PAD commonly affects the lower extremities and can lead to tissue necrosis (gangrene). Home management instructions for PAD include: Lower the extremities below the heart when sitting and lying down - improves arterial blood flow Engage in moderate exercise (eg, 30- to 45-minute walk, twice daily) - promotes collateral circulation and distal tissue perfusion Perform daily skin care, including application of lotion - prevents skin breakdown from dry skin Maintain mild warmth (eg, lightweight blankets, socks) - improves blood flow and circulation Stop smoking - prevents vessel spasm and constriction Avoid tight clothing and stress - prevents vasoconstriction Take prescribed medications (eg, vasodilators, antiplatelets) - increases blood flow and prevents blood clot development Educational objective:Peripheral artery disease increases the risk of tissue necrosis and limb loss. Management focuses on improving blood flow and circulation to the extremities through lifestyle changes and medications.

The nurse observes the rhythm shown in the exhibit on a client's cardiac monitor. The client reports palpitations and lightheadedness. Which intervention does the nurse anticipate? Click on the exhibit button for additional information. 1. Adenosine IVP 2. Atropine IVP 3. Defibrillation 4. External pacing

1. Adenosine IVP This client is experiencing paroxysmal supraventricular tachycardia (PSVT). In PSVT, the heart rate can be 150-220/min. With prolonged episodes, the client may experience evidence of reduced cardiac output such as hypotension, palpitations, dyspnea, and angina. Treatment includes vagal maneuvers such as Valsalva, coughing, and carotid massage. Adenosine is the drug of choice for PSVT treatment. Due to its very short half-life, adenosine is administered rapidly via IVP over 1-2 seconds and followed by a 20-mL saline bolus. An increased dose may be given twice if previous administration is ineffective. Beta blockers, calcium channel blockers, and amiodarone can also be considered as alternatives. If vagal maneuvers and drug therapy are unsuccessful, synchronized cardioversion may be used. Educational objective:The drug of choice in clients with PSVT is adenosine. It is given rapidly via IVP over 1-2 seconds and followed by a 20-mL saline bolus. An increased dose may be administered 2 more times if previous administration is ineffective.

The nurse is caring for a client with cardiomyopathy and coronary artery disease. The client is reporting increasing chest pain and has bilateral lung crackles on auscultation. The health care provider has written several new prescriptions. Which new prescription should the nurse clarify? Select all that apply. Vital signs Blood pressure 84/58 mm Hg Heart rate 108/min Respirations 28/min Oxygen saturation 90% 1. Administer 2,000 mL normal saline bolus 2. Administer IV nitroglycerin 3. Apply 4 L oxygen by nasal cannula 4. Obtain a STAT 12-lead ECG 5. Obtain blood for cardiac enzyme testing

1. Administer 2,000 mL normal saline bolus 2. Administer IV nitroglycerin Cardiomyopathy is a group of diseases in which the heart muscle (ie, myocardium) has a reduced ability to pump blood effectively, placing clients at risk for cardiogenic shock. Cardiogenic shock is manifested by reduced cardiac output (eg, hypotension, narrow pulse pressure), which can lead to pulmonary edema (eg, tachypnea, bibasilar crackles, decreased oxygen saturation) caused by blood "backing up" into the pulmonary capillaries. To compensate, catecholamines (eg, epinephrine) and vasopressin are released by the adrenal glands to increase cardiac output. However, this compensatory mechanism eventually fails, causing decreased perfusion and oxygenation of tissues as well as death. The client may need additional support with ionotropic agents (eg, norepinephrine) in these situations. Supplemental oxygen is appropriate for treatment given low oxygen saturation, chest pain, and tachypnea (Option 3). If chest pain is present, obtaining an ECG and testing cardiac enzymes (eg, creatine kinase-MB, troponin I) are appropriate (Options 4 and 5). Educational objective:Clients with cardiomyopathy may develop cardiogenic shock due to the heart's inability to circulate blood effectively, causing reduced cardiac output. Treatment of cardiogenic shock includes supplemental oxygen, an ECG, cardiac enzyme testing, and interventions to reduce cardiac workload.

A client comes to the emergency department for the second time with shortness of breath and substernal pressure that radiates to the jaw. The nurse understands that angina pectoris may be precipitated by which of these factors? Select all that apply. 1. Amphetamine use 2. Cigarette smoking 3. Cold exposure 4. Deep sleep 5. Sexual intercourse

1. Amphetamine use 2. Cigarette smoking 3. Cold exposure 5. Sexual intercourse Angina pectoris is defined as chest pain brought on by myocardial ischemia (decreased blood flow to the heart muscle). Any factor that increases oxygen demand or decreases oxygen supply to cardiac muscle may cause angina, including the following: Physical exertion (eg, exercise, sexual activity): Increases heart rate and reduces diastole (time of maximum blood flow to the myocardium) Intense emotion (eg, anxiety, fear): Initiates the sympathetic nervous system and increases cardiac workload Temperature extremes: Usually cold exposure and hypothermia (vasoconstriction); occasionally hyperthermia (vasodilation and blood pooling) Tobacco use and second-hand smoke inhalation: Replaces oxygen with carbon monoxide; nicotine causes vasoconstriction and catecholamine release Stimulants (eg, cocaine, amphetamines): Increase heart rate and cause vasoconstriction Coronary artery narrowing (eg, atherosclerosis, coronary artery spasm): Decreases blood flow to myocardium Educational objective:Angina pectoris is chest pain caused by myocardial ischemia. Any factor that increases oxygen demand or decreases oxygen supply may deprive the myocardium of necessary oxygen needed to function effectively.

The nurse is caring for a client who just had aortic valve replacement surgery. Which assessment information is most important to report to the health care provider (HCP)? 1. Chest tube output of 175 mL in past hour 2. International Normalized Ratio (INR) of 1.5 3. Temperature of 100.3 F (37.9 C) 4. Total urine output of 85 mL over past 3 hours

1. Chest tube output of 175 mL in past hour Chest drainage >100 mL/hr should be reported to the HCP. Large losses of blood may indicate a compromise of the surgical suture site and may require repair. The client can quickly become hemodynamically unstable and may require a return to surgery or transfusion of blood products. Educational objective:Postoperative blood loss >100 mL/hr should be reported to the HCP immediately. The client may have a compromised suture site and can rapidly become hemodynamically unstable.

The nurse is preparing to administer medications after assessing a client with a myocardial infarction. Based on the collected data, which of the following prescribed medications are appropriate for the nurse to administer? Click on the exhibit button for additional information. Select all that apply. Vital signs at 0800 Temperature98.4 F (36.9 C) Blood pressure126/81 mm Hg Heart rate49/min Respirations16/min 1. Aspirin 2. Atorvastatin 3. Docusate sodium 4. Lisinopril 5. Metoprolol

1. Aspirin 2. Atorvastatin 3. Docusate sodium 4. Lisinopril Myocardial infarctions (MIs) damage heart muscle and require medications to improve heart function and prevent reinfarction (eg, aspirin). Aspirin, an antiplatelet agent, inhibits platelet aggregation, prevents thrombus formation, and reduces heart inflammation. Clients without signs of bleeding or low platelet levels may safely receive aspirin (Option 1). Atorvastatin is a lipid-lowering medication given to clients to lower cholesterol levels (ie, LDL cholesterol), which reduces plaque and reinfarction risk (Option 2). However, statins may cause rhabdomyolysis and require monitoring for muscle weakness and pain. Docusate sodium is a stool softener that reduces straining during bowel movements, thereby decreasing the workload on the heart. Straining can also cause bradycardia due to vagal response (Option 3). Lisinopril is an ACE inhibitor often prescribed to clients after an MI to prevent ventricular remodeling and progression of heart failure. Lisinopril may cause hyperkalemia and hypotension, and should be administered only to clients with normokalemia and normotension (Option 4). Educational objective:Nurses should use clinical data and assessment to determine prescription safety. Beta blockers require monitoring of heart rate and blood pressure. ACE inhibitors require monitoring of potassium and blood pressure. Aspirin requires monitoring of platelet levels and signs of bleeding. Statins require monitoring for muscle pain.

The unlicensed assistive personnel reports a client blood pressure of 90/60 mm Hg to the nurse. The client's prescriptions say to notify the health care provider (HCP) if systolic blood pressure is <100 mm Hg. What should the nurse do first? 1. Assess the client for other signs and symptoms 2. Immediately notify the client's HCP 3. Notify the charge nurse on duty for the shift 4. Review the client's medication administration record

1. Assess the client for other signs and symptoms The nurse should follow the client's prescriptions and notify the HCP, after first assessing for any other signs and symptoms that may be associated with the low blood pressure (eg, dizziness, pallor, signs of poor perfusion, confusion). The HCP will most likely ask the nurse about the type of symptoms as well as past vital signs, medications, and laboratory results. The nurse should utilize SBAR, a common tool for communication with HCPs that includes Situation, Background, Assessment, and Recommendation. The nurse should have all of this information on hand prior to calling the HCP. Educational objective:The nurse should have current assessment data and access to the client's recent laboratory data, diagnostic studies, and medication administration record before calling the HCP.

The nurse is caring for a client with end-stage heart failure. The rhythm shown in the exhibit is seen on the cardiac monitor, and the nurse finds the client unresponsive with no palpable pulse. What is the correct interpretation of this rhythm? Click on the exhibit button for additional information. 1. Asystole 2. Complete heart block 3. Disconnected lead wire 4. Pulseless electrical activity

1. Asystole Asystole is characterized by no electrical activity or obvious wave. Clients will have no pulse or respirations, and will be unresponsive (Option 1). Clients with advanced cardiac disease or heart failure are at increased risk for developing asystole. Cardiopulmonary resuscitation (CPR) should be initiated, followed by advanced cardiac life support measures, including administration of epinephrine, placement of an advanced airway, and treatment of any reversible causes. Educational objective:Asystole is characterized by complete absence of electrical activity on the ECG. The client will have no pulse or respirations, and will be unresponsive. The nurse should immediately initiate cardiopulmonary resuscitation, advanced cardiac life support measures, and treatment of any reversible causes.

The nurse is caring for a client who just had a permanent ventricular pacemaker inserted. The nurse observes the cardiac monitor and sees a pacing spike followed by a QRS complex for each heartbeat. How should the nurse assess for mechanical capture of the pacemaker? 1. Auscultate the client's apical pulse rate 2. Measure the client's blood pressure 3. Obtain a 12-lead ECG 4. Palpate the client's radial pulse rate

1. Auscultate the client's apical pulse rate Clients with an implanted permanent pacemaker should be assessed for both electrical capture of heart rhythm and mechanical capture of heart rate. In atrial pacing, pacer spikes precede P waves, whereas in ventricular pacing, pacer spikes precede QRS complexes. Pacing spikes should be immediately followed by their appropriate electrical waveform, indicating electrical capture. Checking for mechanical capture is essential to ensure that the electrical activity of the heart corresponds to a pulsatile rhythm. The best method for checking for a pulsatile rhythm is to assess a central pulse (eg, auscultation of apical, palpation of femoral) (Option 1). This rate should be compared to the electrical rate displayed on the cardiac monitor to assess for pulse deficit. Educational objective:For clients with a newly implanted permanent pacemaker, the nurse should assess for electrical capture of heart rhythm (eg, ECG) and mechanical capture of heart rate (eg, pulse). A central pulse (eg, auscultation of apical, palpation of femoral) should be assessed to determine mechanical capture.

A client diagnosed with heart failure has an 8-hour urine output of 200 mL. What is the nurse's first action? 1. Auscultate the client's breath sounds 2. Encourage the client to increase fluid intake 3. Report the findings to the health care provider (HCP) 4. Start an intravenous line for diuretic administration

1. Auscultate the client's breath sounds Urine output of less than 30 mL/hr may indicate low vascular volume (dehydration, blood loss), decreased renal perfusion (low cardiac output), intrinsic kidney injury, or urine outflow obstruction (enlarged prostate, kinked Foley catheter). Given this client's heart failure, low urine output is likely due to decreased cardiac function and buildup of fluid in the lungs. The nurse should assess the lung sounds for crackles and report to the HCP, who can prescribe loop diuretics. Educational objective:Decreased urine output of <30 mL/hr could be due to low vascular volume (dehydration, blood loss), decreased renal perfusion (low cardiac output), intrinsic kidney injury, or urine outflow obstruction (enlarged prostate, kinked Foley catheter). Always assess the client first, and then report to the HCP.

The nurse identifies which risk factors as contributing to the development of peripheral artery disease? Select all that apply. 1. Cigarette smoking 2. Diabetes mellitus 3. Hyperlipidemia 4. Oral contraceptive use 5. Prolonged standing

1. Cigarette smoking 2. Diabetes mellitus 3. Hyperlipidemia Educational objective: In peripheral artery disease, arteries in the extremities become atherosclerotic (progressive thickening and hardening due to chronic damage). Peripheral tissue perfusion is impaired, causing pain with exercise (eg, intermittent claudication) and at rest. Risk factors include hypertension, diabetes mellitus, hyperlipidemia, and smoking.

The nurse is planning discharge teaching for a client who just received a permanent pacemaker. Which topics should the nurse include? Select all that apply. 1. Avoid MRI scans 2. Do not place cell phones directly over the pacemaker 3. Notify airport security when traveling 4. Perform shoulder range-of-motion exercises 5. Refrain from using microwave ovens

1. Avoid MRI scans 2. Do not place cell phones directly over the pacemaker 3. Notify airport security when traveling Discharge teaching for the client with a permanent pacemaker should include the following: Report fever or any signs of redness, swelling, or drainage at the incision site. Carry a pacemaker identification card and wear a medical alert bracelet. Take the pulse daily and report it to the health care provider (HCP) if below the predetermined rate. Avoid MRI scans, which can affect or damage a pacemaker (Option 1). Avoid carrying a cell phone in a pocket directly over the pacemaker and, when talking on a cell phone, hold it to the ear on the opposite side of the pacemaker (Option 2). Notify airport security of a pacemaker; a handheld screening wand should not be held directly over the device (Option 3). Avoid standing near antitheft detectors in store entryways; walk through at a normal pace and do not linger near the device. Educational objective:Clients with permanent pacemakers should carry a pacemaker identification card, wear a medical alert bracelet, avoid MRI scans, avoid placing a cell phone over the pacemaker, and inform airport security personnel. Above-the-shoulder exercises should be avoided on the side of the pacemaker until cleared by the health care provider. Microwave ovens are safe to use.

The nurse is teaching a client diagnosed with Raynaud phenomenon about ways to prevent recurrent episodes. Which instructions should the nurse include? Select all that apply. 1. Avoid excessive caffeine 2. Immerse hands in cold water 3. Practice yoga or tai chi 4. Refrain from using tobacco products 5. Wear gloves when handling cold objects

1. Avoid excessive caffeine 3. Practice yoga or tai chi 4. Refrain from using tobacco products 5. Wear gloves when handling cold objects Raynaud phenomenon is a vasospastic disorder resulting in an episodic vascular response related to cold temperatures or emotional stress. It most commonly affects women age 15-40. Vasospasms induce a characteristic color change in the appendages (eg, fingers, toes, ears, nose). When vasoconstriction occurs, the affected appendage initially turns white from decreased perfusion, followed by a bluish-purple appearance due to cyanosis. Clients usually report numbness and coldness during this stage. When blood flow is subsequently restored, the affected area becomes reddened and clients experience throbbing or aching pain, swelling, and tingling. Acute vasospasms are treated by immersing the hands in warm water. Client teaching regarding prevention of vasospasms includes: Wear gloves when handling cold objects (Option 5). Dress in warm layers, particularly in cold weather. Avoid extremes and abrupt changes in temperature. Avoid vasoconstricting drugs (eg, cocaine, amphetamines, ergotamine, pseudoephedrine). Avoid excessive caffeine intake (Option 1). Refrain from use of tobacco products (Option 4). Implement stress management strategies (eg, yoga, tai chi) (Option 3). If conservative management is unsuccessful, clients may be prescribed calcium channel blockers to relax arteriole smooth muscle and prevent recurrent episodes. Educational objective:Raynaud phenomenon is a vasospastic disorder triggered by exposure to cold or stress. Key elements of client teaching include management of acute attacks, avoidance of vasoconstrictive substances (eg, tobacco, cocaine, caffeine), stress reduction, and appropriate clothing (eg, gloves, warm layers).

The nurse is caring for a client who has been admitted to the hospital for an acute exacerbation of heart failure. Blood pressure is 104/62 mm Hg, pulse is 96/min, respirations are 22/min, and oxygen saturation is 91%. Which of these findings supports the diagnosis of acute heart failure exacerbation? 1. B-type natriuretic peptide (BNP) 1382 pg/mL [1382 pmol/L] 2. Flat jugular veins when seated at a 45-degree angle 3. Sodium 150 mEq/L [150 mmol/L 4. Urine output greater than 100 mL/hr

1. B-type natriuretic peptide (BNP) 1382 pg/mL [1382 pmol/L Brain (or b-type) natriuretic peptide (BNP) is secreted in response to ventricular stretch and wall tension when cardiac filling pressures are elevated. The BNP level is used to differentiate dyspnea of heart failure from dyspnea of noncardiac etiology. The level of circulating BNP correlates with both severity of left ventricular filling pressure elevation and mortality. A normal BNP level is <100 pg/mL [<100 pmol/L]. The nurse would expect a high BNP in a client exhibiting symptoms of acute decompensated heart failure. (Option 2) Jugular veins should normally flatten and disappear as the client is raised to an upright position. Jugular venous distension present above a 45-degree seated position indicates fluid volume excess and elevated cardiac filling pressures that occur with heart failure. Educational objective:The nurse should assess the BNP level in clients admitted with heart failure exacerbations. Elevated BNP levels indicate increased ventricular stretch and correlate with severity of heart failure and fluid volume overload. Heart failure clients may also present with jugular venous distension, low serum sodium, and decreased urine output.

A cardiac catheterization was performed on a client 2 hours ago. The catheter was inserted into the left femoral artery. What signs of potential complications should the nurse report immediately to the health care provider (HCP)? Select all that apply. 1. Bleeding at the catheterization site 2. Client lying down and quietly watching television 3. Client taking only sips of fluids 4. Left foot remarkably cooler than right foot 5. Urine output of 100 mL since the procedure

1. Bleeding at the catheterization site 4. Left foot remarkably cooler than right foot Bleeding at the puncture site indicates that a clot has not formed at the insertion site. This is an arterial bleed as catheterization was done via the femoral artery. Arterial bleeds can lead to hypovolemic shock and death if not treated immediately. Reduced warmth in the lower extremity of the insertion site is a sign of decreased perfusion (lack of oxygenated blood flow) to the extremity and can result in tissue necrosis of the affected area. Educational objective:If not treated immediately, arterial bleeds can lead to hypovolemic shock and death. Reduced warmth in the lower extremity of the catheter insertion site is a sign of decreased perfusion (lack of oxygenated blood flow).

The nurse cares for a client who had an abdominal aortic aneurysm repair 6 hours ago. Which assessment would require immediate follow-up? 1. Abdomen is soft, nondistended, and tender to touch 2. Blood pressure is 96/66 mm Hg and apical pulse is 112/min 3. Client rates pain as 4 on a scale of 0-10(0%) 4. Green bile is draining from the nasogastric tube

2. Blood pressure is 96/66 mm Hg and apical pulse is 112/min Abdominal aortic aneurysms are surgically repaired when they measure about 6 cm or are causing symptoms. Repair can be done via femoral percutaneous placement of a stent graft (endovascular aneurysm repair) or via an open surgical incision of the aneurysm with synthetic graft placement. The client must be monitored postoperatively for graft leakage and hemodynamic stability. Adequate blood pressure is necessary to maintain graft patency, and prolonged hypotension can lead to the formation of graft thrombosis. Signs of graft leakage include a decreasing blood pressure and increasing pulse rate. Educational objective:Following repair of an abdominal aortic aneurysm, hemodynamic stability is a priority. Prolonged hypotension can lead to graft thrombosis. A falling blood pressure and rising pulse rate can also signify graft leakage.

The nurse is preparing to administer 40 mg of IV furosemide. Prior to administering the medication, the nurse should assess which parameters? Select all that apply. 1. Blood pressure 2. Blood urea nitrogen 3. Liver enzymes 4. Potassium 5. White blood cell count

1. Blood pressure 2. Blood urea nitrogen 4. Potassium Loop diuretics (furosemide, torsemide, bumetanide) are used to treat fluid retention, such as that found in clients with heart failure or cirrhosis. When administering loop diuretics, the nurse can expect the client's kidneys to excrete a significant amount of water and potassium. When potassium is excreted at a fast rate, the client could develop hypokalemia, a medical emergency that can result in other life-threatening complications such as heart arrythmias, as well as muscle cramps and weakness (Option 4). Blood pressure should also be assessed prior to administration of loop diuretics as excess diuresis may cause intravascular volume depletion that results in low blood pressure. A client with baseline hypotension may develop a critically low blood pressure. Excess diuresis can also affect kidneys, and the blood urea nitrogen and creatinine levels can become elevated as well. Therefore, these levels should be assessed (Options 1 and 2). Educational objective:When administering furosemide, it is important to closely monitor the client's vital signs, serum electrolytes (potassium), and kidney function tests (blood urea nitrogen, creatinine) prior to administration to prevent side effects such as hypokalemia, hypotension, and kidney injury.

A client with suspected moderate to large pericardial effusion is admitted for monitoring. The nurse performs a head-to-toe assessment. Which of these findings indicate likely cardiac tamponade and require immediate intervention? Select all that apply. 1. Blood pressure of 90/70 mm Hg 2. Bounding peripheral pulses 3. Decreased breath sounds on left side 4. Distant heart tones 5. Jugular venous distension

1. Blood pressure of 90/70 mm Hg 4. Distant heart tones 5. Jugular venous distension Pericardial effusion is a buildup of fluid in the pericardium. Tamponade, a serious complication of pericardial effusion, develops as the effusion increases in volume and results in compression of the heart. The heart struggles to contract effectively against the fluid, and cardiac output can decrease drastically. This life-threatening complication requires an emergency pericardiocentesis (a needle inserted into the pericardial sac to remove fluid). Signs and symptoms of cardiac tamponade include: Hypotension with narrowed pulse pressure (Option 1) Muffled or distant heart tones (Option 4) Jugular venous distension (Option 5) Pulsus paradoxus Dyspnea, tachypnea Tachycardia Educational objective:The client with a moderate to large pericardial effusion is at risk for the development of cardiac tamponade. Signs and symptoms of tamponade include muffled or distant heart tones, narrowed pulse pressure, jugular venous distension, pulsus paradoxus, dyspnea, tachypnea, and tachycardia. The nurse should report these findings to the health care provider immediately and prepare for a pericardiocentesis.

Which clinical finding would the nurse anticipate in a client with chronic venous insufficiency? 1. Brownish, hardened skin on lower extremities 2. Diminished peripheral pulses 3. Nonhealing ulcer on lateral surface of great toe 4. Shiny, hairless lower extremities

1. Brownish, hardened skin on lower extremities Chronic venous insufficiency (CVI) occurs when the valves in the veins of the lower extremities consistently fail to keep venous blood moving forward, which causes chronic increased venous pressure. The increased pressure pushes fluid out of the vascular space and into the surrounding tissues, where tissue enzymes break down red blood cells. The destruction of red blood cells releases hemosiderin (a reddish-brown protein that stores iron), which causes a brownish skin discoloration; chronic edema and inflammation cause the tissue to harden and appear leathery (Option 1). Affected skin is highly prone to breakdown and ulcerations (eg, venous leg ulcers), commonly on the inside of the ankle. Educational objective:Chronic venous insufficiency occurs when the valves in the veins of the lower extremities fail to keep blood moving forward. Chronic edema and inflammatory changes lead to brownish, thickened skin on the extremities and venous leg ulcers (commonly on the inside of the ankle).

A client with severe vomiting and diarrhea has a blood pressure of 90/70 mm Hg and pulse of 120/min. IV fluids of 2-liter normal saline were administered. Which parameters indicate that adequate rehydration has occurred? Select all that apply. 1. Capillary refill is less than 3 seconds 2. Pulse pressure is narrowed 3. Systolic blood pressure drops only when standing 4. Urine output is 360 mL in 4 hours 5. Urine specific gravity is 1.020

1. Capillary refill is less than 3 seconds 4. Urine output is 360 mL in 4 hours 5. Urine specific gravity is 1.020 This client's initial vital signs show tachycardia and hypotension, which are classic signs of hypovolemia. Normal capillary refill is less than 3 seconds and is an indication of normal hydration and perfusion (Option 1). Obligatory urine output is 30 mL/hr, and this client has 90 mL/hr. Urine output is one of the best indicators of adequate rehydration (Option 4). The urine specific gravity is within a normal range (1.003 to 1.030), which can indicate normal hydration (Option 5). Educational objective:Signs of adequate hydration are normal urine specific gravity (1.003 to 1.030), adequate volume of urine output (>30 mL/hr), and capillary refill of less than 3 seconds. Pulse pressure narrows in shock, and positive orthostatic vital signs (decreasing systolic blood pressure and rising heart rate) with position change indicate dehydration

The nurse is caring for a client after percutaneous placement of a coronary stent for a myocardial infarction. The client rates lower back pain as 5 on a scale of 0-10 and has blood pressure of 140/92 mm Hg. The cardiac monitor shows normal sinus rhythm with occasional premature ventricular contractions. Which prescription should the nurse administer first? Potassium 3.3 mEq/L (3.3 mmol/L) Sodium 149 mEq/L (149 mmol/L) Glucose157 mg/dL (8.7 mmol/L). 1. Captopril PO every 8 hours 2. Morphine IV PRN for pain 3. Potassium chloride IVPB once 4. Regular insulin subcutaneous with meals

1. Captopril PO every 8 hours Clients with myocardial infarction (MI) are at risk for life-threatening dysrhythmias (eg, heart block, ventricular tachycardia, ventricular fibrillation) both during the MI and following reperfusion therapy (eg, coronary artery stenting). Myocardial ischemia damages cardiac muscle cells, causing electrical irritability (eg, premature ventricular contractions) that can be exacerbated by electrolyte imbalances (eg, hypokalemia). Hypokalemia hyperpolarizes cardiac electrical conduction pathways, increasing the risk for dysrhythmias. Therefore, prompt potassium replacement is the priority in these clients (Option 3). Educational objective:Prompt potassium replacement is the priority action for hypokalemic clients with myocardial infarction because they are at increased risk for life-threatening dysrhythmias (eg, heart block, ventricular tachycardia, ventricular fibrillation) and cardiac arrest.

The nurse receives hand-off report on assigned clients. Which client should the nurse assess first? 1. Client 1 day post femoral-popliteal bypass surgery who now has a nonpalpable pedal pulse present only with Doppler 2. Client with chronic venous insufficiency who has edema and brown discoloration of the lower extremities 3. Client with peripheral arterial disease and gangrene of the foot who has a cool-to-the-touch, hairless extremity 4. Client with peripheral arterial disease who reports severe cramping pain in the calf with activity such as walking

1. Client 1 day post femoral-popliteal bypass surgery who now has a nonpalpable pedal pulse present only with Doppler Femoral-popliteal bypass surgery involves circumventing a blockage in the femoral artery with a synthetic or autogenous (artery or vein) graft to restore blood flow. The nurse performs neurovascular assessments on the affected extremity (ie, pulses, color and skin temperature, capillary refill, pain, movement) and compares the findings with the preoperative baselines. The client's nonpalpable pedal pulse that is present only with Doppler distal to the graft (ie, post-tibial, pedal) can indicate compromised blood flow or graft occlusion and should be reported to the health care provider immediately. (Option 2) Chronic venous insufficiency is the inability of the leg veins to efficiently pump blood back to the heart. It can lead to venous stasis, increased hydrostatic pressure, and venous leg ulcers. Edema and thick skin with brown pigmentation are expected manifestations, so this is not the priority assessment. Educational objective:Absent or decreased volume in the peripheral pulses distal to the graft can indicate compromised circulation or graft occlusion and should be reported to the health care provider immediately.

What clinical symptoms might the nurse expect to find in a client with a central venous pressure (CVP) of 24 mm Hg? Select all that apply. 1. Crackles in lungs 2. Dry mucous membranes 3. Hypotension 4. Jugular venous distension 5. Pedal edema

1. Crackles in lungs 4. Jugular venous distension 5. Pedal edema CVP is a measurement of right ventricular preload (volume within the ventricle at the end of diastole) and reflects fluid volume problems. The normal CVP is 2-8 mm Hg. An elevated CVP can indicate right ventricular failure or fluid volume overload. Clinical signs of fluid volume overload include the following: Peripheral edema Increased urine output that is dilute Acute, rapid weight gain Jugular venous distension S3 heart sound in adults Tachypnea, dyspnea, crackles in lungs Bounding peripheral pulses Educational objective:Elevated CVP can indicate fluid volume overload. The nurse should recognize clinical signs such as crackles in lungs, jugular venous distension, and peripheral edema as evidence of fluid volume overload.

A client is hospitalized with worsening chronic heart failure. Which clinical manifestations does the admitting nurse most likely assess in this client? Select all that apply. 1. Crackles on auscultation 2. Dry mucous membranes 3. Increased jugular venous distention 4. Rhonchi on auscultation 5. Skin "tenting" 6. 3+ pitting edema of the lower extremities

1. Crackles on auscultation 3. Increased jugular venous distention 5. Skin "tenting" 6. 3+ pitting edema of the lower extremities Clients with a diagnosis of chronic congestive heart failure experience clinical manifestations of both right-sided (systemic venous congestion) and left-sided (pulmonary congestion) failure. Crackles are discontinuous, adventitious lung sounds usually heard on inspiration and indicate the presence of pulmonary congestion (left-sided failure) in this client. Increased jugular venous distention reflects an increase in pressure and volume in the systemic circulation, resulting in elevated central venous pressure (CVP) (right-sided failure) in this client. Although dependent pitting edema of the extremities can be associated with other conditions (eg, hypoproteinemia, venous insufficiency), it is related to sodium and fluid retention (right-sided failure) in this client. Educational objective:Clients with chronic heart failure experience clinical manifestations of both right-sided and left-sided failure. Therefore, the nurse must be able to assess for the clinical manifestations related to systemic volume increases and pulmonary congestion.

The nurse is preparing to discharge a client who developed heart failure after a myocardial infarction. Based on the discharge data, the nurse plans to include which topics during teaching?Vital signs Temperature 98.2 F ( 36.7 C ) Blood pressure 108/72 mm Hg Heart rate 62/min Respirations 16/min SpO2 96% on room air 1. Daily weighing 2. How to take own pulse 3. Need for monthly International Normalized Ratio (INR) 4. Need to increase foods high in potassium 5. Reduction of sodium in diet 6. Use of home oxygen

1. Daily weighing 2. How to take own pulse 5. Reduction of sodium in diet This client with heart failure would need to measure weight daily, restrict sodium and fluid intake, and know how to take a pulse. Educational objective:The client being discharged with heart failure should receive teaching related to weight monitoring, diet, medication regimen, activity, and symptoms to report.

A client with a blood pressure (BP) of 250/145 mm Hg is admitted for hypertensive crisis. The health care provider prescribes a continuous IV infusion of nitroprusside sodium. Which of these is the priority goal in initial management of hypertensive crisis? 1. Decrease mean arterial pressure (MAP) by no more than 25% 2. Keep blood pressure at or below 120/80 mm Hg 3. Maintain heart rate (HR) of 60-100/min 4. Maintain urine output of at least 30 mL/hr

1. Decrease mean arterial pressure (MAP) by no more than 25% Hypertensive crisis is a life-threatening emergency due to the possibility of severe organ damage. If not treated promptly, complications such as intracranial hemorrhage, heart failure, myocardial infarction (MI), renal failure, aortic dissection, or retinopathy may occur. Emergency treatment includes IV vasodilators such as nitroprusside sodium. It is important to lower the blood pressure slowly, as too rapid a drop may cause decreased perfusion to the brain, heart, and kidneys. This may result in stroke, renal failure, or MI. The initial goal is usually to decrease the MAP by no more than 25% or to maintain MAP at 110-115 mm Hg. The pressure can then be lowered further over a period of 24 hours. MAP is calculated by adding the systolic blood pressure (SBP) and double the diastolic blood pressure (DBP), and then dividing the resulting value by 3. MAP = (2 x DBP + SBP) / 3 Educational objective:Hypertensive crisis may require continuous infusion of an IV vasodilator. BP should be lowered slowly to prevent organ damage. The initial goal is to lower MAP by 25% or less or to maintain MAP of 110-115 mm Hg.

The nurse cares for a client who had an abdominal aortic aneurysm repair 6 hours ago. Which assessment findings would indicate possible graft leakage and require a report to the primary care provider? Select all that apply. 1. Ecchymosis of the scrotum 2. Increased abdominal girth 3. Increased urinary output 4. Report of groin pain 5. Report of increased thirst and appetite loss

1. Ecchymosis of the scrotum 2. Increased abdominal girth 4. Report of groin pain xplanation Explanation: Repair of abdominal aortic aneurysms can be done via femoral percutaneous placement of a stent graft (endovascular aneurysm repair) or via an open surgical incision of the aneurysm and placement of a synthetic graft. With either procedure, postoperative monitoring for graft leakage or separation is a priority. Manifestations of graft leakage include ecchymosis of the groin, penis, scrotum, or perineum; increased abdominal girth; tachycardia; weak or absent peripheral pulses; decreasing hematocrit and hemoglobin; increased pain in the pelvis, back, or groin; and decreased urinary output (Options 1, 2, and 4). Educational objective:Signs of graft leakage that are important to monitor after repair of an abdominal aortic aneurysm include pain in the back, pelvis, or groin; ecchymosis of the groin, scrotum, or penis; tachycardia; weak or absent peripheral pulses; decreasing hematocrit and hemoglobin; increased abdominal girth; and decreased urinary output.

A nurse is teaching the parents of an infant with tetralogy of Fallot. Which of the following actions should the nurse include to reduce the incidence of hypercyanotic spells? Select all that apply. 1. Encourage smaller, frequent feedings 2. Offer a pacifier when the infant begins to cry 3. Promote a quiet period upon waking in the morning 4. Swaddle the infant during procedures 5. Turn the infant frequently during sleep

1. Encourage smaller, frequent feedings 2. Offer a pacifier when the infant begins to cry 3. Promote a quiet period upon waking in the morning 4. Swaddle the infant during procedures Additional: Educational objective: Hypercyanotic or tet spells usually occur during stressful or painful procedures; on waking; and with hunger, crying, and feeding. Providing a calm environment; reducing hunger with small, frequent meals; and swaddling during procedures can help prevent hypercyanotic spells.

The nurse is providing discharge instructions to the parent of a child with Kawasaki disease. The nurse informs the parent that the presence of which symptom should be immediately reported to the health care provider? 1. Fever 2. Irritability 3. Knee pain 4. Skin peeling

1. Fever Kawasaki disease (KD) is a systemic vasculitis of childhood that presents with ≥5 days of fever, nonexudative conjunctivitis, lymphadenopathy, mucositis, hand and foot swelling, and a rash. First-line treatment consists of IV immunoglobulin and aspirin to prevent coronary artery aneurysms. When children with KD are discharged home, parents are instructed to monitor them for fever by checking the temperature (orally or rectally) every 6 hours for the first 48 hours following the last fever. Temperature should also be checked daily until the follow-up appointment. If the child develops a fever, the health care provider should be notified as this may indicate the acute phase of KD recurrence. The child may require additional treatment with IV immunoglobulin to prevent development of coronary artery aneurysms and occlusions. Educational objective: Once children with KD are discharged home, parents should be instructed to check their temperature every 6 hours for the first 48 hours following the last fever and then daily until the follow-up visit. The health care provider should be notified if the child has fever as this may indicate a need for further treatment.

The nurse is assessing for the presence of jugular venous distension (JVD) on a newly admitted client with a history of heart failure. Which is the best position for the nurse to place the client in when observing for JVD? 1. Head of the bed elevated to a 45-degree angle 2. Head of the bed elevated to a 60-degree angle 3. Head of the bed elevated to a 90-degree angle 4. Head of the bed flat

1. Head of the bed elevated to a 45-degree angle Distension of jugular neck veins should be performed with the client sitting with the head of the bed at a 30- to 45-degree angle. The nurse will observe for distension and prominent pulsation of the neck veins. The presence of JVD in the client with heart failure may indicate an exacerbation and possible fluid overload. Educational objective:The nurse should position the client with the head of the bed at a 30- to 45-degree angle to assess for the presence of JVD.

The registered nurse (RN) is providing discharge instructions to a client who has had coronary artery bypass grafting (CABG). Which teaching is correct? Select all that apply. 1. No sexual activity for at least 6 weeks postoperatively 2. Notify health care provider (HCP) of redness, swelling, or drainage at the incision site 3. Refrain from lifting objects weighing >5 lb (2.26 kg) until approved by the HCP 4. Take a shower daily without soaking chest and leg incisions 5. Use lotion on incision sites with dressing changes if the area is dry

1. No sexual activity for at least 6 weeks postoperatively 2. Notify health care provider (HCP) of redness, swelling, or drainage at the incision site 3. Refrain from lifting objects weighing >5 lb (2.26 kg) until approved by the HCP 4. Take a shower daily without soaking chest and leg incisions 5. Use lotion on incision sites with dressing changes if the area is dry The RN providing discharge instructions for a client recovering from a CABG should include the following guidelines: Explain the need for modification of cardiac risk factors, including smoking cessation, weight reduction, maintaining a healthy diet, and increasing activity levels through exercise. Encourage a daily shower (Option 4) as a bath could introduce microorganisms into the surgical incision sites. Surgical incisions are washed gently with mild soap and water and patted dry. The incisions should not be soaked or have lotions or creams applied as this could introduce pathogens (Option 5). Explain that light house work may begin in 2 weeks, but there is to be no lifting of any object weighing >5 lb (2.26 kg) without approval of the HCP (Option 3). Lifting, carrying, and pushing heavy objects are isometric activities. Heart rate and blood pressure increase rapidly during isometric activities, which should be limited until approved by the HCP, generally about 6 weeks after discharge. Guide the client to gradually resume activity and possibly participate in a cardiac rehabilitation program. Clarify no driving for 4-6 weeks or until the HCP approves. If the client is able to walk 1 block or climb 2 flights of stairs without symptoms (eg, chest pain, shortness of breath, fatigue), it is usually safe to resume sexual activity (Option 1). Notify the HCP if the following symptoms occur:Chest pain or shortness of breath that does not subside with restFever >101 F (38.3 C)Redness, drainage, or swelling at the incision sites (Option 2). Educational objective:Discharge teaching for a client recovering from a CABG should include instructions related to medications, activity level, driving, sexual activity, and symptoms to report to the HCP.

The nurse provides discharge instructions to a client who was hospitalized for deep venous thrombosis (DVT) that is now resolved. Which of the following instructions should the nurse include to prevent the reoccurrence of DVT? Select all that apply. 1. "Do not take car rides longer than 4 hours for at least 3-4 weeks." 2. "Drink plenty of fluids every day and limit caffeine and alcohol intake." 3. "Elevate legs on a footstool when sitting and dorsiflex the feet often." 4. "Resume your walking program as soon as possible after getting home." 5. "Sit in a cross-legged position for 5-10 minutes to improve circulation."

2. "Drink plenty of fluids every day and limit caffeine and alcohol intake." 3. "Elevate legs on a footstool when sitting and dorsiflex the feet often." 4. "Resume your walking program as soon as possible after getting home." A deep venous thrombosis (DVT) is a blood clot (ie, thrombus) formed in large veins, generally of the lower extremities. Risk factors for DVT include venous stasis, blood hypercoagulability, and endothelial damage. Therefore, discharge teaching for a client with resolved DVT emphasizes interventions to promote blood flow and venous return (eg, exercise, smoking cessation) to prevent reoccurrence. Interventions to prevent DVT reoccurrence include: Obtain adequate fluid intake and limit caffeine and alcohol intake to avoid dehydration because dehydration increases the risk for blood hypercoagulability (Option 2). Elevate the legs when sitting and dorsiflex the feet often to reduce edema and promote venous return (Option 3). Resume an exercise program (eg, walking, swimming) and change positions frequently to promote venous return (Option 4). Stop smoking to prevent endothelial damage and vasoconstriction. Avoid restrictive clothing (eg, tight jeans), which interferes with circulation and promotes clotting. Consult with a dietitian if overweight; excess weight increases venous insufficiency by compressing large pelvic vessels. Educational objective:Discharge teaching for a client with resolved deep vein thrombosis includes interventions to prevent reoccurrence (eg, take in adequate fluids, elevate the extremities, exercise regularly, change positions frequently, stop smoking)

A clinic nurse is caring for a client who has hypertension and is prescribed hydrochlorothiazide, lisinopril, and clonidine. The current blood pressure reading is 190/102 mm Hg, and the client reports a headache that has lasted several days. Which question is most important for the nurse to ask next? 1. "Have you noticed any abnormal swelling in your legs?" 2. "How are you currently taking your blood pressure medications?" 3. "How has your stress level been the past few weeks?" 4. "What over-the-counter medications have you taken today?

2. "How are you currently taking your blood pressure medications?" A major problem in the long-term management of hypertension is poor adherence to the treatment plan, often due to unpleasant side effects (eg, fatigue, dizziness, reduced libido, erectile dysfunction) and medication cost. This problem can worsen if a client must take multiple medications. Determining whether a client is taking medications as prescribed is a priority, as sudden or abrupt discontinuation of antihypertensive medications can cause rebound hypertension and possibly hypertensive crisis (eg, blurred vision, dizziness, severe headache, shortness of breath) (Option 2). Educational objective:A major problem in long-term management of hypertension is poor adherence to the treatment plan, often due to unpleasant side effects and medication cost. Assessing for medication adherence is important, as abrupt discontinuation of antihypertensive medications can cause rebound hypertension and hypertensive crisis.

A client is being discharged after receiving an implantable cardioverter defibrillator. Which statement by the client indicates that teaching has been effective? 1. "I'm not worried about the device firing now because I know it won't hurt." 2. "I will let my daughter fix my hair until my health care provider says I can do it." 3. "I will look into public transportation because I won't be able to drive again." 4. "I will notify my travel agent that I can no longer travel by plane."

2. "I will let my daughter fix my hair until my health care provider says I can do it." An implantable cardioverter defibrillator (ICD) can sense and defibrillate life-threatening dysrhythmias. It also includes pacemaker capabilities such as overdrive pacing for rapid heart rhythms or back-up pacing for bradycardias that may occur after defibrillation. The ICD consists of a lead system placed into the endocardium via the subclavian vein. The pulse generator is implanted subcutaneously over the pectoral muscle. Postoperative care and teaching are similar to those for pacemaker implantation. Clients are instructed to refrain from lifting the affected arm above the shoulder (until approved by the health care provider) to prevent dislodgement of the lead wire on the endocardium (Option 2). (Option 1) Firing of the ICD may be painful. Clients have described the feeling as a blow to the chest. (Option 3) Driving may be approved by the health care provider after healing has occurred. Long-term decisions are based on the ongoing presence of dysrhythmias, frequency of ICD firings, and state laws regarding drivers with ICDs. (Option 4) Travel is not restricted. The ICD may set off the metal detector in security areas. A hand-held wand may be used but should not be held directly over the ICD. The client should carry the ICD identification card and a list of medications while traveling. Educational objective:After placement of an implantable cardioverter defibrillator, clients are instructed to avoid lifting the arm on the side of the ICD above the shoulder (until cleared by the health care provider) to avoid dislodging the lead wire system.

A client comes to the emergency department in acute decompensated heart failure. The client is very anxious, with a respiratory rate of 30/min and pink, frothy sputum. After placing the client on oxygen via nasal cannula, which of these actions is the next priority? 1. Administer digoxin 0.25 mg 2. Administer furosemide 40 mg IV push 3. Initiate dopamine infusion at 5 mcg/kg/min 4. Obtain blood sample for arterial blood gases

2. Administer furosemide 40 mg IV push This client is exhibiting signs of pulmonary edema, a life-threatening condition. In the presence of acute left ventricular failure, pulmonary vasculature overload causes increased pulmonary venous pressure that forces fluid out of the vascular space into the pulmonary interstitium and, if untreated, into the alveoli. Clinical manifestations of pulmonary edema include: A history of orthopnea and/or paroxysmal nocturnal dyspnea Anxiety and restlessness Tachypnea (often >30/min), dyspnea, and use of accessory muscles Frothy, blood-tinged sputum Crackles on auscultation The priority of care is to improve oxygenation by reducing pulmonary pressure and congestion. Diuretics (eg, furosemide) are prescribed to remove excess fluid in pulmonary edema (Option 2). Management of acute decompensated heart failure (ADHF) may also include oxygen therapy, vasodilators (eg, nitroglycerin, nesiritide), and positive inotropes (eg, dopamine, dobutamine). Vasodilators decrease preload thus improving cardiac output and decreasing pulmonary congestion. Positive inotropes improve contractility but are only recommended if other medications have failed or in the presence of hypotension. Educational objective:In the presence of acute decompensated heart failure (ADHF) and pulmonary edema, diuretic (eg, furosemide) administration is effective in removing excess fluid to reduce pulmonary congestion and improve oxygenation. Vasodilators (eg, nitroglycerin, nesiritide) and positive inotropes (eg, dopamine, dobutamine) are also used in the treatment of ADHF.

A 62-year-old client is scheduled for open abdominal aneurysm repair. What key assessment should be made by the nurse preoperatively? 1. Assess and compare blood pressure in each arm 2. Assess character and quality of peripheral pulses 3. Assess for presence or absence of hair on lower extremities 4. Assess for presence of bowel sounds

2. Assess character and quality of peripheral pulses Open aneurysm repair involves a large abdominal incision and requires cross-clamping the aorta proximally and distally to the aneurysm. Establishing baseline data is essential for comparison with postoperative assessments. The nurse should pay special attention to the character and quality of peripheral pulses and renal and neurologic status. Dorsalis pedis and posterior tibial pulse sites should be marked for easy location postoperatively. A decreased or absent pulse with cool, pale, mottled, or painful extremity postoperatively can indicate embolization or graft occlusion. Graft occlusion may require reoperation. Educational objective:Preoperative assessment of the character and quality of peripheral pulses provides a baseline for rapid postoperative assessment and identification of emergent complications (embolization, graft occlusion).

The nurse is admitting a client with heart failure-related fluid overload. Which action should the nurse complete first? 1 Administer oxygen 2. Assess the client's breath sounds 3. Initiate cardiac monitoring 4. Insert a peripheral IV catheter

2. Assess the client's breath sounds The client being admitted for heart failure-related fluid overload is likely to have dyspnea (difficulty breathing), orthopnea (labored breathing in a supine position), and paroxysmal nocturnal dyspnea (waking suddenly with difficulty breathing). The assessment phase of the nursing process must come before intervention and should be prioritized using the ABCs: airway, breathing, and circulation. Therefore, the nurse should first assess the client's breath sounds (Option 2). Rales or "crackles" may be auscultated in the lungs as a result of pulmonary congestion. Educational objective:The client with heart failure-related fluid overload usually has respiratory symptoms. Assessment of the client's breathing is the greatest priority. It should be performed before assessment of cardiac rhythm and interventions such as oxygen and placing an IV for diuretic administration.

The nurse is inspecting the legs of a client with a suspected lower-extremity deep venous thrombosis. Which of the following clinical manifestations should the nurse expect? Select all that apply. 1. Blue, cyanotic toes 2. Calf pain 3. Dry, shiny, hairless skin 4. Lower leg warmth and redness 5. Unilateral leg edema

2. Calf pain 4. Lower leg warmth and redness 5. Unilateral leg edema A deep venous thrombosis (DVT) is a blood clot (ie, thrombus) formed in large veins, generally of the lower extremities. DVTs occur commonly as a result of decreased activity or mobility (eg, prolonged travels, bed rest) or as a complication of hospitalization or surgery. Although clients with a DVT may have no symptoms, typical clinical manifestations include unilateral edema, localized pain (eg, calf pain) or tenderness to touch, warmth, erythema, and occasionally low-grade fever (Options 2, 4, and 5). Recognition of a potential DVT is critical because the thrombus can dislodge from the vessel and cause life-threatening pulmonary embolism. Educational objective:A deep venous thrombosis (DVT) is a blood clot formed in large veins, typically of the lower extremities, that occurs commonly from decreased activity or mobility. Clinical manifestations of a lower-extremity DVT include unilateral edema, calf pain or tenderness to touch, warmth, erythema, and low-grade fever.

A client is in suspected shock state from major trauma. Which parameters best indicate the adequacy of peripheral perfusion? Select all that apply. 1. Apical pulse 2. Capillary refill 3. Lung sounds 4. Pupillary response 5. Skin color and temperatur

2. Capillary refill 5. Skin color and temperature Shock is a life-threatening syndrome characterized by decreased perfusion and impaired cellular metabolism. A lack of perfusion at both the tissue and cellular level (anaerobic metabolism) occurs due to decreased cardiac output, ineffective blood flow, and inability to meet the body's demand for increased oxygen. Sustained hypoperfusion activates compensatory mechanisms (eg, neural, hormonal, biochemical) to maintain homeostasis and reverse the consequences of anaerobic metabolism. Shock will progress through 4 stages (initial, compensatory, progressive, irreversible). Early identification and intervention help to prevent stage progression. Adequacy of tissue perfusion in a client with shock syndrome and possible organ dysfunction is assessed by the level of consciousness, urine output, capillary refill, peripheral sensation, skin color, extremity temperature, and peripheral pulses. Capillary refill indicates adequacy of blood flow to the peripheral tissues. It is measured by the time taken for color (pink) to return to an external capillary bed (nail bed) after pressure is applied to cause blanching. In an adult, color should return in less than 3 seconds. Normal skin color and temperature are indicators of the adequacy of peripheral blood flow; these are usually within normal limits during the initial and compensatory stages of shock. . Educational objective:The adequacy of blood flow to peripheral tissues is determined by measuring capillary refill and assessing skin color and temperature; these are usually within normal limits during the initial and compensatory stages of shock.

Which interventions should the nurse include when caring for a client who has had endovascular repair of an abdominal aortic aneurysm? Select all that apply. 1. Assess abdominal incision every 4 hours 2. Check for bleeding at groin puncture sites 3. Measure chest tube drainage 4. Monitor fluid intake and urine output 5. Palpate and monitor peripheral pulses

2. Check for bleeding at groin puncture sites 4. Monitor fluid intake and urine output 5. Palpate and monitor peripheral pulses ation Explanation: Endovascular abdominal aortic aneurysm repair is a minimally invasive procedure that involves the placement of a sutureless aortic graft inside the aortic aneurysm via the femoral artery. It does not require an abdominal incision. The nurse will need to monitor the puncture sites in the groin area for bleeding or hematoma formation (Option 2). Peripheral pulses should be palpated and monitored frequently in the early post-op period and routinely afterward (Option 5). Renal artery occlusion can occur due to graft migration or thrombosis so careful monitoring of urine output and kidney function should be part of nursing care (Option 4). Educational objective:The nurse needs to monitor groin puncture sites, peripheral pulses, urine output, and kidney function in the client who has had minimally invasive endovascular repair of an abdominal aneurysm.

The nurse in the outpatient clinic is reviewing phone messages. Which client should the nurse call back first? 1. Client post kidney transplant who reports white spots in the oral cavity 2. Client with a history of mitral valve regurgitation who reports fatigue 3. Client with erythema and purulent drainage at the site of a spider bite 4. Client with hypertension who reports a cold and nasal congestion

2. Client with a history of mitral valve regurgitation who reports fatigue Mitral valve regurgitation is the result of a disrupted papillary muscle(s) or ruptured chordae tendineae, allowing a backflow of blood from the left ventricle through the mitral valve into the left atrium. This backflow can lead to dilation of the left atrium, reduced cardiac output, and pulmonary edema. Clients are often asymptomatic but are instructed to report any new symptoms indicative of heart failure (eg, dyspnea, orthopnea, weight gain, cough, fatigue). This client should be assessed first due to possible heart failure, which would require immediate intervention. Educational objective:Chronic mitral valve regurgitation is often asymptomatic, but many clients eventually develop heart failure; therefore, early recognition of symptoms is a priority. Mitral regurgitation causes a backflow of blood from the left ventricle to the left atrium, resulting in pulmonary edema (eg, dyspnea, orthopnea) and decreased cardiac output (eg, fatigue). Left atrial enlargement can also result in atrial fibrillation (eg, palpitations).

A client is diagnosed with a small thoracic aortic aneurysm during a routine chest x-ray and follows up 6 months later with the health care provider (HCP). Which assessment data is most important for the nurse to report to the HCP? 1. Blood pressure (BP) of 140/86 mm Hg 2. Difficulty swallowing 3. Dry, hacking cough 4. Low back pain

2. Difficulty swallowing Difficulty swallowing is the most important symptom to report to the HCP. A thoracic aortic aneurysm can put pressure on the esophagus and cause dysphagia. The development of this symptom may indicate that the aneurysm has increased in size and may need further diagnostic evaluation and treatment. Educational objective: The nurse should report swallowing difficulty immediately in a client with a thoracic aortic aneurysm. This could indicate that the aneurysm has increased in size and may require treatment.

The nurse is caring for a client with acute pericarditis. Which clinical finding would require immediate intervention by the nurse? 1. Client reports chest pain that is worse with deep inspiration 2. Distant heart tones and jugular venous distension 3. ECG showing ST-segment elevations in all leads 4. Pericardial friction rub auscultated at the left sternal border

2. Distant heart tones and jugular venous distension Acute pericarditis is inflammation of the membranous sac (pericardium) surrounding the exterior of the heart, which can cause an increase in the amount of fluid in the pericardium (ie, pericardial effusion). Increased pericardial fluid places pressure on the heart, which impairs the heart's ability to contract and eject blood. This complication (ie, cardiac tamponade) is life-threatening without immediate intervention. When assessing clients with pericarditis, it is critical for the nurse to observe for signs of cardiac tamponade (eg, muffled or distant heart tones, hypotension, jugular venous distension) (Option 2). Development of cardiac tamponade requires emergency pericardiocentesis (ie, needle insertion into the pericardium to remove fluid) to prevent cardiac arrest. Educational objective:Nurses caring for clients with pericarditis should monitor for, and immediately report, signs of cardiac tamponade (eg, jugular venous distension, distant heart sounds, hypotension), a life-threatening complication occurring from increased pericardial fluid volume.

A nurse auscultates a loud cardiac murmur on a newborn with suspected trisomy 21 (Down syndrome). A genetic screen and an echocardiogram are scheduled that day. The neonate's vital signs are shown in the exhibit. What would be an appropriate action for the nurse to complete next? Vital signs Temperature 98.6 F (37 C) Heart rate 146/min Respirations 42/min O2 saturation 98% 1. Call the health care provider (HCP) immediately 2. Document the assessment finding 3. Place the neonate in a knee-chest position 4. Provide oxygen to the neonate

2. Document the assessment finding Atrioventricular (AV) canal defect is a cardiac anomaly often associated with trisomy 21 (Down syndrome). As an echocardiogram is already scheduled for that day, documenting the assessment finding would be the appropriate action for the nurse to complete at this time. Educational objective: Trisomy 21 (Down syndrome) is often associated with the cardiac anomaly AV canal defect. Assessment typically includes a loud murmur that requires no immediate action when vital signs are stable. Surgery will correct the anomaly when the neonate grows in size and can tolerate the invasive procedure better.

The nurse is admitting a client with a diagnosis of right-sided heart failure resulting from pulmonary hypertension. What clinical manifestations are most likely to be assessed? Select all that apply. 1. Crackles in lung bases 2. Increased abdominal girth 3. Jugular venous distension 4. Lower extremity edema 5. Orthopnea

2. Increased abdominal girth 3. Jugular venous distension 4. Lower extremity edema Right-sided heart failure results from pulmonary hypertension, right ventricular myocardial infarction, or left-sided heart failure. The right ventricle cannot effectively pump blood to the lungs, which results in incomplete emptying of the right ventricle. The resulting decrease in forward blood flow causes blood to back up into the right atrium and then into venous circulation, resulting in venous congestion and increased venous pressure throughout the systemic circulation. Clinical manifestations of right-sided heart failure include: Peripheral and dependent edema (eg, sacrum, legs, hands), especially in the lower extremities (Option 4). Jugular venous distension (Option 3). Increased abdominal girth due to venous congestion of the gastrointestinal tract (eg, hepatomegaly, splenomegaly) and ascites. Nausea and anorexia may also occur as a result of increased abdominal pressure and decreased gastrointestinal circulation (Option 2). Hepatomegaly due to hepatic venous congestion. Educational objective:In clients with right-sided heart failure, the heart cannot effectively pump blood to the lungs. Clinical manifestations result from systemic venous congestion and include peripheral edema, jugular venous distension, increased abdominal girth (hepatomegaly, splenomegaly), and ascites.

The clinic nurse is providing instructions to a client who will be wearing a Holter monitor for the next 24 hours. Which instructions are important to review with the client? 1. How to transmit the readings over the phone 2. Keep a diary of activities and any symptoms experienced 3. Refrain from exercising while wearing the monitor 4. The monitor may be removed only when bathing

2. Keep a diary of activities and any symptoms experienced A Holter monitor continuously records a client's electrocardiogram rhythm for 24-48 hours. Electrodes are placed on the client's chest and a portable recording unit is kept with the client. At the end of the prescribed period, the client returns the unit to the health care provider's (HCP) office. The data can then be recalled, printed, and analyzed for any abnormalities. Client instructions include the following: Keep a diary of activities and any symptoms experienced while wearing the monitor so that these may later be correlated with any recorded rhythm disturbances Do not bathe or shower during the test period (Option 4) Engage in normal activities to simulate conditions that may produce symptoms that the monitor can record (Option 3) Educational objective:The nurse should instruct the client with a Holter monitor to keep a diary of activities and any symptoms that occur while wearing it. The client should also be taught not to bathe during the testing period but to continue all other normal activities.

The nurse is performing an initial assessment on a client in hypertensive crisis. What is the nurse's priority assessment? Click on the exhibit button for additional information. 1. Heart sounds 2. Level of consciousness 3. Lung sounds 4. Visual fields and acuity

2. Level of consciousness Hypertensive crisis is a life-threatening medical emergency characterized by severely elevated blood pressure (systolic ≥180 mm Hg and/or diastolic ≥120 mm Hg). The client may have symptoms of hypertensive encephalopathy, including severe headache, confusion, nausea/vomiting, and seizure. Hypertensive crisis poses a high risk for end-organ damage (eg, hemorrhagic stroke, kidney injury, heart failure, papilledema). The nurse should prioritize neurological assessment (eg, level of consciousness [LOC], cranial nerves) as decreased LOC may indicate onset of hemorrhagic stroke, which requires immediate surgical intervention (Option 2). Treatment for hypertensive crisis typically includes IV nitrates or antihypertensives (eg, nitroprusside, labetalol, nicardipine) and continuous monitoring (eg, blood pressure, telemetry, urine output) in a critical care setting. Educational objective:Hypertensive crisis is a life-threatening elevation in blood pressure (systolic ≥180 mm Hg and/or diastolic ≥120 mm Hg) that may cause end-organ damage (eg, stroke, kidney injury, heart failure, papilledema). The client's level of consciousness should be monitored, as a decreased level may indicate onset of hemorrhagic stroke.

The nurse is caring for a newborn with patent ductus arteriosus. Which assessment finding should the nurse expect? 1. Harsh systolic murmur 2. Loud machine-like murmur 3. Soft diastolic murmur 4. Systolic ejection murmur

2. Loud machine-like murmur Patent ductus arteriosus (PDA) is an acyanotic congenital defect more common in premature infants. When fetal circulation changes to pulmonary circulation outside the womb, the ductus arteriosus should close spontaneously. This closure is caused by increased oxygenation after birth. If a PDA is present, blood will shunt from the aorta back to the pulmonary arteries via the opened ductus arteriosus. Many newborns are asymptomatic except for a loud, machine-like systolic and diastolic murmur. The PDA will be treated with surgical ligation or IV indomethacin to stimulate duct closure. Educational objective:The ductus arteriosus of a newborn should close spontaneously when fetal circulation changes to pulmonary circulation. If the ductus arteriosus remains open, blood will shunt from the aorta to the pulmonary arteries. The child will be acyanotic but will have a machine-like murmur heard on both systole and diastole.

The nurse is planning care for a child being admitted with Kawasaki disease and should give priority to which nursing intervention? 1. Apply cool compresses to the skin of the hands and feet 2. Monitor for a gallop heart rhythm and decreased urine output 3. Prepare a quiet, non-stimulating, and restful environment 4. Provide soft foods and liberal amounts of clear liquids

2. Monitor for a gallop heart rhythm and decreased urine output Educational objective: Kawasaki disease causes inflammation of the arterial walls and can lead to scarring of the coronary arteries or development of coronary aneurysms. Treatment consists of aspirin and substantial infusion of IV gamma globulin. The affected child must be monitored for signs of heart failure.

The nurse is performing a cardiac assessment on a client. The nurse auscultates a loud blowing sound at the second intercostal space, right sternal border. How should the nurse document this finding? 1. Arterial bruit 2. Murmur heard at the aortic area 3. Pericardial friction rub 4. S3 gallop heard at the mitral area

2. Murmur heard at the aortic area Murmurs are produced by turbulent blood flow across diseased or malformed cardiac valves. They can be characterized as musical, blowing, swooshing, or rasping sounds heard between normal heart sounds. The aortic area is located at the second intercostal space, right sternal border. Educational objective:Murmurs indicate turbulent blood flow across diseased or malformed cardiac valves. They are often described as musical, blowing, or swooshing sounds that occur between normal heart sounds. They may be auscultated at the aortic, pulmonic, tricuspid, or mitral areas.

The nurse is caring for a client who had a large anterior wall myocardial infarction (MI) 24 hours ago. Which finding is most important to report to the health care provider (HCP)? 1. Nausea and vomiting 2. New S3 heart sound 3. Occasional unifocal premature ventricular contractions (PVCs) 4. Temperature of 100.4 F

2. New S3 heart sound A large anterior wall MI can affect the pumping ability of the left ventricle, putting the client at risk for developing heart failure and cardiogenic shock. The new development of pulmonary congestion on x-ray, auscultation of a new S3 heart sound, crackles on auscultation of breath sounds, or jugular venous distension can signal heart failure and should be reported immediately to the HCP. Educational objective:The nurse should immediately report the new development of pulmonary congestion on x-ray, auscultation of a new S3 heart sound, crackles on auscultation of breath sounds, or jugular venous distension in the post MI client to the HCP. These findings may indicate the development of heart failure or cardiogenic shock.

An experienced registered nurse (RN) is mentoring a new nurse in the telemetry unit. Which assessment technique by the new nurse requires intervention by the RN? 1. Nurse carefully auscultates for heart murmurs at Erb's point 2. Nurse palpates bilateral carotid arteries simultaneously to assess for symmetry 3. Nurse places client in semi-Fowler's position to assess for jugular venous distension 4. Nurse positions client supine to assess the point of maximal impulse

2. Nurse palpates bilateral carotid arteries simultaneously to assess for symmetry The pulses in the neck should be palpated for information on arterial blood flow. The carotid arteries should be palpated separately to avoid vagal stimulation causing dysrhythmias such as bradycardia or a syncopal episode. Pulse symmetry for other key arteries (eg, temporal, brachial, radial, posterior tibial) is assessed by bilaterally palpating each pair simultaneously. Educational objective:The nurse should not palpate the carotid arteries simultaneously due to possible vagal stimulation resulting in bradycardia or syncope. Each carotid artery should be palpated separately.

A female client with liver cirrhosis and chronic anemia is hospitalized for a deep venous thrombosis. The client is receiving a heparin infusion and suddenly develops epistaxis. Which laboratory value would indicate that the heparin infusion needs to be turned off? 1. Hematocrit of 30% (0.30) 2. Partial thromboplastin time of 110 seconds 3. Platelet count of 80,000/mm3 (80 x 109/L) 4. Prothrombin time of 11 seconds

2. Partial thromboplastin time of 110 seconds Heparin is an anticoagulant that helps prevent further clot formation. It is titrated based on a partial thromboplastin time (PTT). The therapeutic PTT target is 1.5-2.0 times the normal reference range of 25-35 seconds. A PTT value >100 seconds would be considered critical and could result in life-threatening side effects. Common sentinel events that result from heparin drips include epistaxis, hematuria, and gastrointestinal bleeds. Educational objective:Heparin infusions require close monitoring by the nurse. The partial thromboplastin time is the laboratory value required to accurately monitor the therapeutic effects of heparin.

The nurse is preparing to perform cardioversion in a client in supraventricular tachycardia shown in the exhibit that has been unresponsive to drug therapy. The client has become hemodynamically unstable. Which step is most important in performing cardioversion? Click on the exhibit button for additional information. 1. Charge the defibrillator 2. Push the synchronize button 3. Sedate the client 4. Select energy level

2. Push the synchronize button The synchronizer switch must be turned on when cardioversion is planned. The synchronize circuit in the defibrillator is programmed to deliver a shock on the R wave of the QRS complex on the electrocardiogram (ECG). This allows the unit to sense this client's rhythm and time the shock to avoid having it occur during the T wave. A shock delivered during the T wave could cause this client to go into a more lethal rhythm (eg, ventricular tachycardia, ventricular fibrillation). If this client becomes pulseless, the synchronize function should be turned off and the nurse should proceed with defibrillation. Synchronized cardioversion is indicated for ventricular tachycardia with a pulse, supraventricular tachycardia, and atrial fibrillation with a rapid ventricular response. Educational objective:To perform safe cardioversion, the synchronizer button must be activated prior to discharging the unit. The synchronizer function allows the unit to sense the client's rhythm and not deliver a shock during a vulnerable time that could cause the client to go into a more lethal rhythm.

A client with deep vein thrombosis (DVT) is receiving a continuous infusion of unfractionated heparin. The client asks the nurse what the heparin is for. How should the nurse respond? 1. "Heparin is a blood thinner that will help to dissolve the clot in your leg." 2. "Heparin will help stabilize the clot in your leg and prevent it from breaking off and traveling to your lungs." 3. "Heparin will keep the current clot from getting bigger and help prevent new clots from forming." 4. "I'm sorry. This is something that your health care provider (HCP) can answer better upon arriving."(

3. "Heparin will keep the current clot from getting bigger and help prevent new clots from forming." Venous thrombosis involves the formation of a thrombus (clot) and the inflammation of the vein. Anticoagulant therapy such as heparin does not dissolve the clot. The clot will be broken down by the body's intrinsic fibrinolytic system over time. The heparin slows the time it takes blood to clot, thereby keeping the current clot from growing bigger and preventing new clots from forming. Educational objective:The nurse should teach the client that the purpose of unfractionated heparin infusion in the treatment of DVT is to slow the time it takes blood to clot, thereby keeping the current clot from getting bigger and preventing new clots from forming.

An elderly client tells the nurse "I have experienced leg pain for several weeks when I walk to the mailbox each afternoon, but it goes away once I stop walking." What is the priority assessment the nurse should perform? 1. Assess for dry, scaly skin on the lower legs 2. Assess for presence or absence of hair growth on lower extremities 3. Check for presence and quality of posterior tibial and dorsalis pedis pulses 4. Obtain a dietary history

3. Check for presence and quality of posterior tibial and dorsalis pedis pulses This client is exhibiting symptoms of intermittent claudication or ischemic muscle pain that can be due to peripheral artery disease (PAD). PAD impairs circulation to the client's extremities. The nurse should first check for the adequacy of blood flow to the lower extremities by palpating for the presence of posterior tibial and dorsalis pedis pulses and their quality. Poor circulation to the extremities can place the client at increased risk for development of arterial ulcers and infection. The quality of circulation to the extremities will guide the treatment plan for this client; management will include risk factor modification for cardiovascular disease, drug therapy, and possibly surgical revascularization. Educational objective:The nurse caring for a client with intermittent claudication from PAD should assess the adequacy of circulation to the extremities by palpating and assessing the quality of posterior tibial and dorsalis pedis pulses. The quality of circulation will guide the treatment plan including risk factor modification, drug therapy, and possible surgical revascularization.

client is being discharged with a prescription for apixaban after being treated for a pulmonary embolus. Which clinical data is most concerning to the nurse? 1. Client eats a vegetarian diet 2. Client has chronic atrial fibrillation 3. Client takes indomethacin for osteoarthritis 4. Client's platelet count is 176 x103/mm3 (176 x109/L)

3. Client takes indomethacin for osteoarthritis A pulmonary embolism (PE) occurs when the pulmonary arteries are blocked by a thrombus. Initial management of PE includes low-molecular-weight heparin (eg, enoxaparin, dalteparin) or unfractionated IV heparin. Once the PE is resolved, maintenance drug therapy often includes oral anticoagulants such as factor Xa inhibitors (eg, apixaban, rivaroxaban, dabigatran). Anticoagulants place the client at increased risk of bleeding, and the nurse should provide education regarding signs and symptoms of bleeding (eg, bruising; blood in the urine; black, tarry stools) and bleeding precautions (eg, use of an electric razor and soft-bristled toothbrush). Concurrent NSAID use (eg, indomethacin, ibuprofen, meloxicam) significantly increases the risk of bleeding. The nurse should discuss this risk with the health care provider prior to initiation of apixaban therapy (Option 3). Educational objective:Maintenance drug therapy after a pulmonary embolus typically includes administration of oral anticoagulants such as factor Xa inhibitors (eg, apixaban). NSAIDs (eg, indomethacin) increase the risk of bleeding when used concurrently with apixaban therapy. The nurse should question initiation of apixaban therapy in the context of NSAID use.

The nurse should plan to teach which client about the need for prophylactic antibiotics prior to dental procedures? 1. Client who had a large anterior wall myocardial infarction (MI) with subsequent heart failure 2. Client who had a mitral valvuloplasty repair 3. Client with a mechanical aortic valve replacement 4. Client with mitral valve prolapse with regurgitation

3. Client with a mechanical aortic valve replacement Certain individuals should receive prophylactic antibiotics prior to dental procedures to prevent infective endocarditis (IE). These include the following: Prosthetic heart valve or prosthetic material used to repair heart valve Previous history of IE Some forms of congenital heart diseaseUnrepaired cyanotic congenital defectRepaired congenital defect with prosthetic material or device for 6 months after procedureRepaired congenital defect with residual defects at the site or adjacent to the site of a prosthetic patch or device Cardiac transplantation recipients who develop heart valve disease Educational objective:Clients with any form of prosthetic material in their heart valves or who have unrepaired cyanotic congenital heart defect or prior history of IE should take prophylactic antibiotics prior to dental procedures to prevent development of IE.

The nurse receives handoff of care report on four clients. Which client should the nurse see first? 1. Client with atrial fibrillation who reports feeling palpitations and has an irregular pulse of 122/min 2. Client with liver cirrhosis who reports bleeding from an IV insertion site and has a platelet count of 48,000 mm3 (48 x 109/L) 3. Client with pericarditis whose blood pressure has decreased from 122/70 mm Hg to 98/68 mm Hg over the past hour 4. Client with pneumonia whose white blood cell count has increased from 14,000 mm3 (14 x 109/L) 8 hours ago to 30,000 mm3 (30 x 109/L)

3. Client with pericarditis whose blood pressure has decreased from 122/70 mm Hg to 98/68 mm Hg over the past hour Acute pericarditis is inflammation of the membranous sac (pericardium) surrounding the exterior of the heart, which can cause an increase in the amount of fluid in the pericardium (ie, pericardial effusion). If pericardial effusions accumulate rapidly or are very large, they may compress the heart, altering the mechanics of the cardiac cycle (ie, cardiac tamponade). Cardiac tamponade decreases atrioventricular filling and impairs the heart's ability to contract and eject blood; it is life-threatening without prompt recognition and treatment. Clinical features of cardiac tamponade include hypotension or narrow pulse pressure, muffled heart sounds, and neck vein distension (Beck triad) (Option 3). In addition, pulsus paradoxus (ie, systolic blood pressure decrease >10 mm Hg during inhalation), chest pain, tachypnea, and tachycardia may be present. Educational objective:Cardiac tamponade is a possible complication of acute pericarditis that impairs cardiac output and is life-threatening without immediate intervention. Clinical features of cardiac tamponade include hypotension, muffled heart sounds, and neck vein distension (Beck triad).

When monitoring an infant with a left-to-right-sided heart shunt, which findings would the nurse expect during the physical assessment? Select all that apply. 1. Clubbing of fingertips 2. Cyanosis when crying 3. Diaphoresis during feedings 4. Heart murmur 5. Poor weight gain

3. Diaphoresis during feedings 4. Heart murmur 5. Poor weight gain Left-to-right cardiac shunts (eg, patent ductus arteriosus, atrial septal defect, ventricular septal defect) result in excess blood flow to the lungs. Manifestations include heart murmur, poor weight gain, diaphoresis with exertion, and signs of heart failure.

A client admitted with acute myocardial infarction suddenly displays air hunger, dyspnea, and coughing with frothy, pink-tinged sputum. What would the nurse anticipate when auscultating the breath sounds of this client? 1. Bronchial breath sounds at lung periphery 2. Clear vesicular breath sounds at lung bases 3. Diffuse bilateral crackles at lung bases 4. Stridor in upper airways

3. Diffuse bilateral crackles at lung bases Acute-onset dyspnea and cough productive of pink, frothy sputum indicate severe pulmonary edema, likely a complication from myocardial infarction. Pink sputum results from ruptured bronchial veins due to high back pressure. The mix of blood and airway fluids creates the pink tinge. On assessment, crackles can be heard at the lung bases. Educational objective:Acute-onset dyspnea and cough with frothy, pink-tinged sputum indicate pulmonary edema. Auscultation reveals crackles at the lung bases.

A client with uncontrolled hypertension is prescribed clonidine. What instruction is most important for the clinic nurse to give this client? 1. Avoid consuming high-sodium foods 2. Change positions slowly to prevent dizziness 3. Don't stop taking this medication abruptly 4. Use an oral moisturizer to relieve dry mouth

3. Don't stop taking this medication abruptly Educational objective:Clonidine is a very potent antihypertensive. Abrupt discontinuation can result in serious rebound hypertensive crisis. Other common side effects of clonidine include dizziness, drowsiness, and dry mouth (the 3 Ds). Beta blockers, another class of blood pressure medications, can result in withdrawal symptoms if discontinued suddenly.

A client with heart failure has gained 5 lb (2.26 kg) over the last 3 days. The nurse reviews the client's blood laboratory results. Based on this information, what medication administration does the nurse anticipate? Laboratory results Sodium 126 mEq/L (126 mmol/L) Potassium 4.8 mEq/L (4.8 mmol/L) Calcium 9.0 mg/dL (2.25 mmol/L) 1. 0.45% sodium chloride IV 2. Calcium gluconate 3. Furosemide 4. Sodium polystyrene sulfonate

3. Furosemide In heart failure, cardiac output is reduced because the heart is unable to pump blood adequately. This reduction in cardiac output reduces perfusion to the vital organs, including the kidneys. Decreased renal blood flow triggers the kidneys to activate the renin-angiotensin system as a compensatory mechanism, which increases blood volume by increasing water resorption in the kidneys. This compensatory mechanism results in fluid volume excess and dilutional hyponatremia (more free water than sodium). Dilutional hyponatremia can be treated with fluid restriction, loop diuretics, and ACE inhibitors (eg, lisinopril, captopril). Furosemide works to resolve hyponatremia by promoting free water excretion, allowing for hemoconcentration and increased sodium levels (Option 3). Educational objective:Heart failure is characterized by reduced cardiac output, which can reduce renal blood flow. Reduced renal blood flow activates the renin-angiotensin system, resulting in fluid volume excess and dilutional hyponatremia. Loop diuretics (eg, furosemide) promote free water excretion, allowing for hemoconcentration and increased sodium levels.

An 8-month-old infant is scheduled for a femorally inserted balloon angioplasty of a congenital pulmonic stenosis in the cardiac catheterization laboratory. Which finding should the nurse report to the health care provider that could possibly delay the procedure? 1. Auscultation of a loud heart murmur 2. Infant has been NPO for 4 hours 3. Infant has severe diaper rash 4. Slight cyanosis of the nail beds

3. Infant has severe diaper rash The nurse should report the presence of severe diaper rash in an infant who has an interventional catheterization procedure planned. The rash may delay the procedure due to possible contamination at the insertion site.

A client with a permanent pacemaker with continuous telemetry calls the nurse and reports feeling lightheaded and dizzy. The client's blood pressure is 75/55 mm Hg. What is the nurse's priority action? Click the exhibit button for additional information. 1. Administer atropine 0.5 mg IV 2. Administer dopamine 5 mcg/kg/min IV 3. Initiate transcutaneous pacing 4. Notify the health care provider

3. Initiate transcutaneous pacing The client is experiencing failure to capture from the permanent pacemaker with subsequent bradycardia and hypotension. Failure to capture appears on the cardiac monitor as pacemaker spikes that are not followed by QRS complexes. Pacemaker malfunction may be caused by a failing battery, malpositioned lead wires, or fibrosis at the tip of lead wire(s) preventing adequate voltage for depolarization. This client is symptomatic (eg, hypotension, dizziness) from insufficient perfusion. The nurse's priority is to use transcutaneous pacemaker pads to normalize the heart rate, stabilize blood pressure, and adequately perfuse organs until the permanent pacemaker is repaired or replaced (Option 3). Administer analgesia and/or sedation as prescribed as transcutaneous pacing is very uncomfortable for the client. (Option 1) Atropine is administered to clients with symptomatic bradycardia; however, this client's symptoms are caused by failure to capture. Therefore, obtaining capture via transcutaneous pacing should resolve the client's symptoms. (Option 2) Dopamine is an inotrope used to treat hypotension due to bradycardia. This client is bradycardic and hypotensive due to failure to capture. If hypotension persists after transcutaneous pacing is initiated, an inotrope may be necessary. (Option 4) The health care provider needs to be notified, but the nurse should first use the transcutaneous pacemaker to stabilize the client. Educational objective:Signs and symptoms of a failing pacemaker include failure to capture (pacer spikes without associated QRS complexes) with bradycardia and hypotension. The nurse should use a transcutaneous pacemaker to stabilize the client until the internal pacemaker can be repaired or replaced.

A client admitted to the cardiac care unit with markedly elevated blood pressure and heart failure is receiving a continuous infusion of nitroprusside. Thirty minutes later, the client appears pale with cold, clammy skin and reports being lightheaded. Which is the priority nursing action? 1. Auscultate the client's lungs 2. Check the client's capillary refill 3. Measure the client's blood pressure 4. Review the client's electrocardiogram

3. Measure the client's blood pressure Sodium nitroprusside is a highly potent vasodilator (both venous and arteriolar). Venous dilation reduces preload (volume of blood in ventricles at the end of diastole), and arterial dilation reduces afterload (resistance ventricle must overcome to eject blood during systole). Sodium nitroprusside is commonly used in hypertensive emergencies and for conditions in which blood pressure control is of utmost importance (eg, aortic dissection, acute hypertensive heart failure). Nitroprusside begins to act within 1 minute and can produce a sudden and drastic drop in blood pressure (symptomatic hypotension) if not monitored properly. Therefore, the client's blood pressure should be monitored closely (every 5-10 minutes). This client's lightheadedness and cold clammy skin are likely due to hypotension. Nitroprusside metabolizes to cyanide, and clients with renal disease can occasionally develop fatal cyanide toxicity. Educational objective: Sodium nitroprusside is given as an infusion for the short-term treatment of acute decompensated heart failure, especially in clients with markedly elevated blood pressure. It is a potent vasodilator and reduces preload and afterload. The main adverse effect is symptomatic hypotension, necessitating close monitoring of blood pressure.

The nurse is assessing a 70-year-old client with a long history of type 2 diabetes mellitus for sudden, severe nausea, diaphoresis, dizziness, and fatigue in the emergency department. Which hospital protocol would be the most appropriate to follow initially? 1. Food poisoning 2. Influenza 3. Myocardial infarction 4. Stroke

3. Myocardial infarction Early recognition and treatment of heart attack are critical. Women, the elderly, and clients with a history of diabetes may not have the classic heart attack symptoms of dull chest pain with radiation down the left arm. Instead, they can present with "atypical" symptoms such as nausea, vomiting, belching, indigestion, diaphoresis, dizziness, and fatigue. (Option 1) Taking a careful history and evaluating for any sick contacts would be helpful in identifying food poisoning, but a more important initial step is to assess for a heart attack. (Option 2) A viral infection is a possibility, but fever and myalgia are usually present during an episode of influenza. (Option 4) Early intervention in stroke is also critical, and a neurologic assessment would take place after the acute coronary syndrome algorithm, especially with negative electrocardiography and serum heart enzyme levels. Educational objective:Myocardial infarctions in women, the elderly, and diabetics may have gastrointestinal distress as the main symptom; this needs to be evaluated with the institutional protocol for acute coronary syndrome.

An experienced nurse is mentoring a new registered nurse (RN) on the telemetry unit. The new RN is measuring orthostatic blood pressure (BP) for a client. Which situation would warrant intervention by the experienced nurse? 1. Nurse has client lie supine for 5-10 minutes prior to starting procedure 2. Nurse interprets a decrease in systolic BP by 10 mm Hg as a normal finding 3. Nurse starts by measuring BP and heart rate (HR) with the client standing 4. Nurse takes BP and HR after standing at 1- and 3-minute intervals

3. Nurse starts by measuring BP and heart rate (HR) with the client standing The experienced nurse should intervene if the new RN starts BP measurement with the client in the standing position. Orthostatic BP measurement may be done to detect volume depletion or postural hypotension caused by medications or autonomic dysfunction. Procedure for measurement of orthostatic BP Have the client lie down for at least 5 minutes (Option 1) Measure BP and HR Have the client stand Repeat BP and HR measurements after standing at 1- and 3-minute intervals (Option 4) A drop in systolic BP of ≥20 mm Hg or in diastolic BP of ≥10 mm Hg, or experiencing lightheadedness or dizziness is considered abnormal (Option 2). Educational objective:To measure orthostatic BP, the nurse should have the client lie supine for 5-10 minutes and then measure BP and HR. The nurse should then have the client stand for 1 minute, measure BP and HR, and repeat the measurements at 3 minutes. Findings are significant if the systolic BP drops ≥20 mm Hg or the diastolic BP drops ≥10 mm Hg

The client was diagnosed 6 months ago with hypertension and had a recent emergency department visit for a transient ischemic attack (TIA). The client's blood pressure today is 170/88 mm Hg. What teaching topic is a priority for the nurse to discuss with this client? 1. Decreasing sodium intake 2. Decreasing stress levels at work and home 3. Increasing activity level 4. Taking blood pressure medications as prescribed

4. Taking blood pressure medications as prescribed The priority teaching topic for this client is taking blood pressure medications as prescribed. A major problem with long-term management of hypertension is poor adherence to the treatment plan. Blood pressure medications can have unpleasant side effects, including fatigue, dizziness, and erectile dysfunction. Client may stop taking the medications when they believe their blood pressure has returned to normal range or if medications are expensive. The nurse should determine whether the client has been taking the medications consistently. There may be a need for a dosage change or addition of another medication. The client's blood pressure is not well controlled, and the TIA places this client at a high risk for a stroke. Educational objective:A major problem with long-term management of hypertension is poor adherence to the treatment plan. The nurse should teach the client the importance of taking blood pressure medications as prescribed.

A client comes to the emergency department with crushing, substernal chest pain. Temperature is 98.6 F (37 C), blood pressure is 173/84 mm Hg, pulse is 92/min, and respirations are 24/min. Oxygen saturation is 95% on room air. What is the nurse's next priority action? 1. Attach defibrillator pads to the client's chest 2. Check the lipid profile laboratory results 3. Obtain a 12-lead electrocardiogram (ECG) 4. Prepare to administer a heparin drip

3. Obtain a 12-lead electrocardiogram (ECG) It is very important to rapidly diagnose and treat the client with chest pain and potential myocardial infarction to preserve cardiac muscle. Initial interventions in emergency management of chest pain are as follows: Assess airway, breathing, and circulation (ABCs) Position client upright unless contraindicated Apply oxygen, if the client is hypoxic Obtain baseline vital signs, including oxygen saturation Auscultate heart and lung sounds Obtain a 12-lead electrocardiogram (ECG) Insert 2-3 large-bore intravenous catheters Assess pain using the PQRST method Medicate for pain as prescribed (eg, nitroglycerin) Initiate continuous electrocardiogram (ECG) monitoring (cardiac monitor) Obtain baseline blood work (eg, cardiac markers, serum electrolytes) Obtain portable chest x-ray Assess for contraindications to antiplatelet and anticoagulant therapy Administer aspirin unless contraindicated Educational objective:Nurses must take presenting cardiac symptoms seriously until the cause is determined. Assess airway, breathing, and circulation, and obtain baseline pulse oximetry and vital signs. Then obtain electrocardiogram (ECG) results.

The nurse is caring for a child with Kawasaki disease who is receiving IV immunoglobulin. The child's parent wants to know why this treatment is required. The nurse explains that this therapy is given to: 1. Fight the infection 2. Minimize rash 3. Prevent heart disease 4. Reduce spleen size

3. Prevent heart disease Kawasaki disease (KD), also known as mucocutaneous lymph node syndrome, is characterized by ≥5 days of fever, bilateral nonexudative conjunctivitis, mucositis, cervical lymphadenopathy, rash, and extremity swelling. Coronary artery aneurysms are the most serious potential sequelae in untreated clients, leading to complications such as myocardial infarction and death. Echocardiography is used to monitor these cardiovascular complications. Intravenous immunoglobulin (IVIG) along with aspirin is used to prevent coronary aneurysms and subsequent occlusion. KD is one of the few pediatric illnesses in which aspirin therapy is warranted due to its antiplatelet and anti-inflammatory properties. However, parents should be cautioned about the risk of Reye syndrome. Cardiopulmonary resuscitation should also be taught to parents of children with coronary artery aneurysms. Educational objective:IVIG along with aspirin is the recommended initial treatment for Kawasaki disease, with the primary goal of coronary disease prevention.

A client with mitral valve prolapse (MVP) has been experiencing occasional palpitations, lightheadedness, and dizziness. The health care provider prescribes a beta blocker. What additional teaching should the nurse include for this client? 1. Avoid aerobic exercise 2. Ensure you receive antibiotics prior to dental work 3. Stay well hydrated and avoid caffeine 4. Wear a medical alert bracelet

3. Stay well hydrated and avoid caffeine Clients with MVP may have palpitations, dizziness, and lightheadedness. Chest pain can occur but its etiology is unknown in this client population. It may be a result of abnormal tension on the papillary muscles. Chest pain that occurs in MVP does not typically respond to antianginal treatment such as nitrates. Beta blockers may be prescribed for palpitations and chest pain. Client teaching for MVP includes the following: Adopt healthy eating habits and avoid caffeine as it is a stimulant and may exacerbate symptoms (Option 3) Check ingredients of over-the-counter medications or diet pills for stimulants such as caffeine or ephedrine as they can exacerbate symptoms Reduce stress and avoid alcohol use Educational objective:The nurse should teach the client with MVP to stay hydrated, avoid caffeine and alcohol, exercise regularly, reduce stress, and take beta blockers as prescribed for palpitations and chest pain. Nitrates are usually not effective for chest pain from MVP.

The home care nurse visits the house of an elderly client. Which assessment finding requires immediate intervention? 1. The client cannot remember what was done yesterday 2. The client has a painful red area on the buttocks 3. The client has new dependent edema of the feet 4. The client has strong, foul smelling urine

3. The client has new dependent edema of the feet New onset of dependent edema of the feet could represent congestive heart failure. This is an urgent medical condition that needs prompt evaluation for characteristic signs (eg, weight gain, lung crackles) and treatment. Educational objective:New onset of dependent edema in an elderly client could be due to heart failure; the client needs further assessment for characteristic signs such as lung crackles and increased body weight (fluid retention).

A client is 48 hours post abdominal aneurysm repair. Which assessment by the nurse is cause for greatest concern? 1. Diminished breath sounds in bilateral lung bases 2. Hypoactive bowel sounds in all 4 quadrants 3. Urinary output of 90 mL in the past 4 hours 4. Warm extremities with 1+ bilateral pedal pulses

3. Urinary output of 90 mL in the past 4 hours. Renal perfusion status is monitored closely in a client who has had abdominal aneurysm repair. Hypotension, dehydration, prolonged aortic clamping during surgery, blood loss, or embolization can lead to decreased renal perfusion and potential kidney injury. The nurse should routinely monitor the client's blood urea nitrogen (BUN) and creatinine levels as well as urine output. Urine output should be at least 30 mL/hr. This client should have an output of at least 120 mL of urine in a 4-hour period. Educational objective:The nurse should carefully monitor renal status in a client who has had abdominal aortic aneurysm repair. BUN, creatinine, and urine output should be assessed. Urine output of at least 30 mL/hr is expected.

A client with heart failure is started on furosemide. The laboratory results are shown in the exhibit. The nurse is most concerned about which condition? Click on the exhibit button for additional information. Sodium 134 mEq/L (134 mmol/L) Potassium 3.4 mEq/L (3.4 mmol/L) Chloride 108 mEq/L (108 mmol/L) Magnesium 0.9 mEq/L (0.45 mmol/L) 1. Atrial fibrillation 2. Atrial flutter 3. Mobitz II 4. Torsades de pointes

4. Torsades de pointes Hypomagnesemia (normal: 1.5-2.5 mEq/L [0.75-1.25 mmol/L]) causes a prolonged QT interval that increases the client's susceptibility to ventricular tachycardia. Torsades de pointes is a type of polymorphic ventricular tachycardia coupled with a prolonged QT interval; it is a lethal cardiac arrhythmia that leads to decreased cardiac output and can develop quickly into ventricular fibrillation. The American Heart Association recommends treatment with IV magnesium sulfate. Educational objective:In a client with hypomagnesemia, it is important to assess the QT interval. The client is most at risk for torsades de pointes, a serious complication that can develop quickly into ventricular fibrillation (lethal arrhythmia).

The nurse has just completed discharge teaching for a client who had aortic valve replacement with a mechanical heart valve. Which statement by the client indicates that teaching has been effective? 1. "I'm glad that I can continue taking my Ginkgo biloba." 2. "I will increase my intake of leafy green vegetables." 3. "I will start applying vitamin E to my chest incision after showering." 4. "I will shave with an electric razor from now on."

4. "I will shave with an electric razor from now on." Mechanical prosthetic valves are more durable than biological valves but require long-term anticoagulation therapy due to the increased risk of thromboembolism. The client should be taught ways to reduce the risk of bleeding. Teaching topics for clients on anticoagulants: Take medication at the same time daily Depending on medication, report for periodic blood tests to assess therapeutic effect Avoid any action that may cause trauma/injury and lead to bleeding (eg, contact sports, vigorous teeth brushing, use of a razor blade) (Option 4) Avoid aspirin and nonsteroidal anti-inflammatory drugs (NSAIDs) Limit alcohol consumption Avoid changing eating habits frequently (eg, dramatically increasing intake of foods high in vitamin K such as kale, spinach, broccoli, greens) (Option 2) and do not take vitamin K supplements Consult with health care provider before beginning or discontinuing any medication or dietary/herbal supplement (eg, Ginkgo biloba and ginseng affect blood clotting and may increase bleeding risk) (Option 1) Wear a medical alert bracelet indicating what anticoagulant is being taken Educational objective:Clients who are on anticoagulants should avoid aspirin, NSAIDS, and other over-the-counter or herbal products (eg, Ginkgo biloba) that can increase bleeding risk. They should also avoid behaviors that increase the risk of clotting (eg, eating excess green leafy vegetables).

The nurse is teaching a client who is scheduled to have an inferior vena cava filter inserted via the right femoral vein. Which statement by the client requires further teaching? 1. "I need to make all health care providers aware of my filter before I have body scans." 2. "I need to stay active and avoid crossing my legs for extended periods when I get home." 3. "I should call the health care provider if I develop numbness, tingling, and swelling in my right leg." 4. "It is normal to have some chest or back discomfort for a few days after filter placement."

4. "It is normal to have some chest or back discomfort for a few days after filter placement." An inferior vena cava filter is a device that is inserted percutaneously, usually via the femoral vein. The filter traps blood clots from lower extremity vessels (eg, embolus from deep venous thrombosis) and prevents them from migrating to the lungs and causing a pulmonary embolism (PE). It is prescribed when clients have recurrent emboli or anticoagulation is contraindicated. Clients should be questioned about and report any metallic implants (eg, vascular filters/coils) to the health care team prior to radiologic imaging, specifically MRI (Option 1). Physical activity should be promoted, and clients should avoid crossing their legs to promote venous return from the legs (Option 2). Leg pain, numbness, or swelling may indicate impaired neurovascular status distal to the insertion site and should be reported immediately (Option 3). Educational objective:An inferior vena cava filter traps thrombi migrating from the lower extremities to the lungs. Discharge teaching includes promotion of physical exercise, reporting of symptoms of pulmonary embolism (eg, chest pain, shortness of breath) and impaired lower extremity circulation (eg, pain, numbness), and notification of the health care team prior to MRI.

A nurse is discussing discharge education with a client after his fifth hospitalization for pulmonary edema caused by his congestive heart failure. Which of the following statements indicates that further teaching is required? 1. "I should supplement my potassium intake." 2. "I should weigh myself daily." 3. "Moderate exercise may be helpful in my condition." 4. "Potato chips are an acceptable snack in moderation."

4. "Potato chips are an acceptable snack in moderation." The client is likely dealing with some level of denial regarding his diagnosis of congestive heart failure. Glossing over the importance of salt avoidance is missing an important opportunity to help them avoid further hospitalizations for the same condition. Educational objective:In congestive heart failure, large changes in clients established dietary habits are necessary to avoid the repeated hospitalizations caused by salt overload.

A nurse is caring for a client on the first day postop after having minimally invasive direct coronary artery bypass (MIDCAB) grafting. The client thought that this surgery was supposed to have a much easier recovery and asks the nurse why it is so painful to take deep breaths. What is the best response by the nurse? 1. "I am sorry you have so much pain. I'll go get your pain medication right now." 2. "Let me call the health care provider (HCP) to see if we can increase the dose of your pain medicine." 3. "Take deep breaths while splinting your chest with a pillow, and use your incentive spirometer every 2 hours. This will help your recovery." 4. "The overall recovery time is expected to be shorter, but initial postop pain can actually be higher with MIDCAB because the incisions are made between the ribs."

4. "The overall recovery time is expected to be shorter, but initial postop pain can actually be higher with MIDCAB because the incisions are made between the ribs." MIDCAB does not involve a sternotomy incision or placement on cardiopulmonary bypass. Several small incisions are made between the ribs. A thoracotomy scope or robot is used to dissect the internal mammary artery (IMA) that is used as a bypass graft. Radial artery or saphenous veins may be used if the IMA is not available. Recovery time is typically shorter with these procedures and clients are able to resume activities sooner than with traditional open chest coronary artery bypass graft surgery. However, clients may report higher levels of pain with MIDCAB due to the thoracotomy incisions made between the ribs. Educational objective:The nurse should teach the client that incisional pain from thoracotomy incisions between the ribs may be very painful after MIDCAB surgery. The nurse should encourage the client to take pain medication before the pain is too intense. The client should also be instructed to cough, breathe deeply while splinting the chest with a pillow, and use the incentive spirometer routinely to reduce the incidence of postop complications.

A client in the emergency department is admitted with a diagnosis of rule out myocardial infarction (MI). Which laboratory test should the nurse monitor to determine if the client has had an MI? 1. D-dimer test 2. Low-density lipoprotein (LDL) 3. Myoglobin 4. Troponin

4. Troponin Troponin is a cardiac specific serum marker that is a highly specific indicator of MI and has greater sensitivity and specificity for myocardial injury than creatine kinase (CK) or CK-MB. Serum levels of troponin T and I increase 4-6 hours after the onset of MI, peak at 10-24 hours, and return to baseline in 10-14 days. However, MI is not diagnosed alone by serum cardiac markers. Electrocardiogram findings and client health history along with a history of pain and risk factors are also used to make the diagnosis of MI. Educational objective:The nurse should monitor serum troponin levels when there is suspicion of MI as it is the most sensitive and specific serum cardiac marker.

The nurse is assigned to the following clients. Which client does the nurse assess/identify as being at greatest risk for the development of a deep venous thrombosis (DVT)? 1. A 25-year-old client with abdominal pain who smokes cigarettes and takes oral contraceptives 2. A 55-year-old ambulatory client with exacerbation of chronic bronchitis and hematocrit of 56% 3. A 72-year-old client with a fever who is 2 days post coronary stent placement 4. An 80-year-old client who is 4 days postoperative from repair of a fractured hip

4. An 80-year-old client who is 4 days postoperative from repair of a fractured hip Venous thromboembolism includes both DVT and pulmonary embolism (PE). DVT is the most common form and occurs most often (80%) in the proximal deep veins (iliac, femoral) of the lower extremities. Virchow's triad describes the 3 most common theories behind the pathophysiology of the venous thrombosis: venous stasis, endothelial damage, and hypercoagulability of blood. Risk factors associated with DVT formation include the following: Trauma (endothelial injury and venous stasis from immobility) Major surgery (endothelial injury and venous stasis from immobility) Prolonged immobilization (eg, stroke, long travel) causing venous stasis Pregnancy (induced hypercoagulable state and some venous stasis by the pressure on inferior vena cava) Oral contraceptives (estrogen is thrombotic) Underlying malignancy (cancer cells release procoagulants) Smoking (produces endothelial damage by inflammation) Old age Obesity and varicose veins (venous stasis) Myeloproliferative disorders (increase blood viscosity) The 80-year-old 4-day postoperative client has the most risk factors: orthopedic hip surgery, prolonged period of immobility/inactivity, and advanced age, and is at greatest risk for developing a DVT. Educational objective:DVT is a frequent, often preventable complication of hospitalization, surgery, and immobilization. Factors that increase the risk for developing a DVT include trauma, surgery (especially orthopedic, knee, hip), prolonged immobility/inactivity, oral contraceptives, pregnancy, varicose veins, obesity, smoking, and advanced age.

The nurse evaluates morning laboratory results for several clients who were admitted 24 hours earlier. Which laboratory report requires priority follow-up? 1. Client with chronic obstructive pulmonary disease who has a PaCO2 of 52 mm Hg (6.9 kPa) 2. Client with heart failure who has a brain natriuretic peptide level of 800 pg/mL (800 ng/L) 3. Client with infected pressure ulcer who has a white blood cell count of 13,000/mm3 (13.0 x 109/L) 4. Client with pulmonary embolism who has a partial thromboplastin time of 127 seconds

4. Client with pulmonary embolism who has a partial thromboplastin time of 127 seconds Clients with pulmonary embolism or deep venous thrombosis are treated with anticoagulation. Unfractionated heparin is one such agent, and its efficacy is measured through partial thromboplastin time (PTT) levels. The goal during anticoagulation therapy is a PTT 1.5-2 times the normal reference range of 25-35 seconds. A PTT of 127 seconds is much too prolonged, and spontaneous bleeding could occur. Educational objective:Clients who are receiving heparin infusion for pulmonary embolism or deep venous thrombosis should be maintained with a partial thromboplastin time of 1.5-2 times the reference range used by the laboratory measuring the level. The normal reference range is typically 25-35 seconds.

A 62-year old client was admitted to the telemetry unit after having an acute myocardial infarction 3 days ago. The client reports to the nurse that the left calf is very tender and feels warm to the touch. Which assessment by the nurse is the priority? 1. Ask the client how long the leg has been tender and warm 2. Assess the electrocardiogram (ECG) for any ectopic beats 3. Check vital signs including pulse oximetry(10%) 4. Complete neurovascular assessment on lower extremities

4. Complete neurovascular assessment on lower extremities This client with a tender calf that feels warm to the touch is exhibiting signs and symptoms of a possible deep vein thrombosis (DVT). Additionally, the client has several risk factors for DVT (age >60, being hospitalized and in bed for 3 days). The nurse will need to notify the health care provider (HCP) immediately. However, prior to this, the nurse must perform a thorough assessment of the client to report to the HCP. The priority action by the nurse should include a thorough neurovascular assessment of the extremities, including presence and quality of dorsalis pedis (DP) and posterior tibial (PT) pulses, temperature of the extremities, capillary refill, and circumference measurements of both calves and thighs. Both extremities should be assessed for comparison. Educational objective:The nurse that suspects DVT should perform a thorough neurovascular assessment of the client's extremities, including presence and quality of DP and PT pulses, temperature of extremities, capillary refill, and circumference measurements of both calves and thighs. Both extremities should be assessed for comparison. The findings should be reported immediately to the HCP.

The nurse is monitoring a client following a radiofrequency catheter ablation. The nurse notes that the P waves are not associated with the QRS complexes on the cardiac monitor. Which intervention is most appropriate at this time? Click on the exhibit button for additional information. 1. Call a code and begin chest compressions 2. Call the rapid response team and prepare for cardioversion 3. Document the findings in the chart and continue to monitor 4. Notify the cardiologist and prepare for temporary pacing

4. Notify the cardiologist and prepare for temporary pacing Radiofrequency ablation is performed through transvenous cardiac catheterization to ablate (ie, burn) electrical pathways causing supraventricular or ventricular tachydysrhythmias. Ablation performed near the atrioventricular (AV) node can damage conduction, causing varying degrees of AV block. Third-degree AV block, or complete heart block, occurs when electrical conduction from the atria to the ventricles is blocked, causing decreased cardiac output (eg, dizziness, syncope, mental status changes, heart failure, hypotension, bradycardia). On ECG, third-degree AV block presents as a regular rate and rhythm with disassociated P waves and QRS complexes. This type of AV block requires temporary or permanent pacing to restore electrical conduction and hemodynamic stability. Educational objective:Third-degree AV block results in disassociation of atrial and ventricular contraction due to blocked electrical conduction pathways. Temporary or permanent pacing is necessary to stabilize the client.

The nurse reviews laboratory data for a client admitted to the emergency department with chest pain. Which serum value requires the most immediate action by the nurse? 1. Glucose 200 mg/dL (11.1 mmol/L) 2. Hematocrit 38% (0.38) 3. Potassium 3.4 mEq/L (3.4 mmol/L) 4. Troponin 0.7 ng/mL (0.7 mcg

4. Troponin 0.7 ng/mL (0.7 mcg Serum cardiac markers are proteins released into the bloodstream from necrotic heart tissue after a myocardial infarction (MI). Troponin is a highly specific cardiac marker for the detection of MI. It has greater sensitivity and specificity for myocardial injury than creatine kinase (CK) MB. Serum levels of troponin increase 4-6 hours after the onset of MI, peak at 10-24 hours, and return to baseline after 10-14 days. A troponin value of 0.7 ng/mL (0.7 mcg/L) indicates cardiac muscle damage and should be the priority and immediate focus of the nurse. Normal values: troponin I <0.5 ng/mL (<0.5 mcg/L); troponin T <0.1 ng/mL (<0.1 mcg/L). Educational objective:An elevated troponin value holds the highest priority for intervention when a client is experiencing chest pain. Positive troponin levels are indicative of myocardial injury and require immediate attention by the nurse. Normal values are <0.5 ng/mL (<0.5 mcg/L) for troponin I and <0.1 ng/mL (<0.1 mcg/L) for troponin T.

A client with an implantable cardioverter defibrillator (ICD) develops ventricular tachycardia (VT) with a pulse while admitted to the medical-surgical unit. The ICD fires multiple times without successfully stopping the VT, causing the client to become confused and difficult to rouse. Which action by the nurse is appropriate? 1. Attempt to stimulate a vagal response by having the client cough 2. Deactivate the client's implantable cardioverter defibrillator with an external magnet 3. Obtain a STAT 12-lead ECG to verify the cardiac rhythm 4. Prepare for synchronized cardioversion with the external defibrillator

4. Prepare for synchronized cardioversion with the external defibrillator An implantable cardioverter defibrillator (ICD) is a medical device that is surgically implanted underneath the skin that can sense life-threatening arrhythmia and discharge electrical shocks directly into the cardiac muscle to correct the arrhythmia. Clients typically receive ICDs after a history of sustained or recurrent ventricular tachycardia (VT), including personal or family history of sudden cardiac death and severe heart failure. When caring for a client with an ICD, it is critical that the nurse monitor for ICD firings (eg, client report, observation on cardiac monitors). After firings, the nurse should monitor for resolution of the arrythmia, indications of hemodynamic compromise (eg, hypotension, chest pain, altered mentation), and additional ICD discharges. Occasionally, an ICD may be unable to convert the arrythmia to a hemodynamically stable rhythm and will repeatedly shock the client. If the client experiences repeated ICD shocks without dysrhythmia resolution, the nurse should promptly obtain a manual external defibrillator and initiate measures to prevent hemodynamic instability and cardiac arrest (Option 4). Educational objective:An implantable cardioverter defibrillator (ICD) is a device used to sense life-threatening arrhythmia and discharge electrical shocks to correct the arrhythmia. If a client experiences repeated ICD shocks without dysrhythmia resolution, the nurse should obtain a manual external defibrillator and initiate cardiac life support.

A client develops sinus bradycardia with blood pressure of 90/40 mm Hg and a heart rate of 46/min. Which of the following actions should the nurse take? 1. Give scheduled dose of metoprolol 50 mg orally 2. Instruct client to cough forcefully 3. Place client in reverse Trendelenburg position 4. Prepare to administer atropine 0.5 mg intravenous (IV) push

4. Prepare to administer atropine 0.5 mg intravenous (IV) push Clients with symptomatic bradycardia should be treated with atropine. If atropine is ineffective, transcutaneous pacing or an infusion of dopamine or epinephrine should be considered. Educational objective:The client with symptomatic bradycardia should be treated initially with IV atropine. Transcutaneous pacing or infusion of dopamine or epinephrine may be considered if atropine is ineffective.

A client with chronic heart failure calls the clinic to report a weight gain of 3 lb (1.36 kg) over the last 2 days. Which information is most important for the nurse to ask this client? 1. Diet recall for this current week 2. Fluid intake for the past 2 days 3. Medications and dosages taken over the past 2 days 4. Presence of shortness of breath, coughing, or edema

4. Presence of shortness of breath, coughing, or edema The client with chronic heart failure is at risk for exacerbations that may require hospitalization. The priority for the nurse on the phone is to ascertain if the client is experiencing any physiological symptoms such as shortness of breath, coughing, or edema (Option 4). These could indicate fluid overload. This information can help the nurse direct the client to come in for further assessment, follow a protocol to make changes in medications/dosages, or restrict fluids. Educational objective:The client with chronic heart failure is at risk for exacerbations. Clients should be instructed to report a weight gain of 3 lb (1.36 kg) over 2 days or a 3-5 lb (1.36-2.26 kg) gain over a week. The nurse's priority assessment should be any physiological signs or symptoms of fluid overload.

After a prolonged surgical procedure, the client reports unilateral leg pain. Which client assessment finding is most concerning? 1. Client rates leg pain as "7" 2. Negative Homan sign 3. Prominent varicose veins bilaterally 4. Right calf is 4 cm larger than left calf

4. Right calf is 4 cm larger than left calf Deep venous thrombosis (DVT) is a major concern in clients with unilateral leg pain after prolonged immobilization (eg, air travel, surgery) or those with obesity, pregnancy, or other hypercoagulable states (eg, cancer). Eighty percent of DVTs start in the veins of the calf and move into the popliteal and femoral veins. Classic symptoms include unilateral leg edema, local warmth, erythema, and low-grade fever. Therefore, the swelling in one leg is highly concerning. Educational objective:Classic signs of deep venous thrombosis are unilateral leg swelling, local warmth, erythema, and low-grade fever in a client with obesity or immobility.

A client with chest pain is diagnosed with acute pericarditis by the health care provider. The nurse explains that the pain will improve with which of the following? 1. Coughing and deep breathing 2. Left lateral position 3. Pursed-lip breathing 4. Sitting up and leaning forward

4. Sitting up and leaning forward The most common cause of acute pericarditis is a recent viral infection. It is an inflammation of the visceral and/or parietal pericardium. Pericarditis is characterized by typical pleuritic chest pain that is sharp. It is aggravated during inspiration and coughing. Pain is typically relieved by sitting up and leaning forward. This position reduces pressure on the inflamed parietal pericardium, especially during lung inflation. The pain is different than that experienced during myocardial infarction. Assessment shows a pericardial friction rub (scratchy or squeaking sound). Treatment includes a combination of nonsteroidal anti-inflammatory drugs (NSAIDS) or aspirin plus colchicine. Educational objective:Pericarditis is characterized by typical pleuritic chest pain that is sharp. It is aggravated during inspiration and coughing. Pain is typically relieved by sitting up and leaning forward. Treatment includes a combination of NSAIDs or aspirin plus colchicine.

Which subjective or objective assessment finding would the nurse expect to find in a client with severe aortic stenosis? 1 Bounding peripheral pulses 2. Diastolic murmur 3. Loud second heart sound 4. Syncope on exertion

4. Syncope on exertion Aortic stenosis is a narrowing of the aortic valve, which obstructs blood flow from the left ventricle to the aorta. As stenosis progresses, the heart cannot overcome the worsening obstruction, and ejects a smaller fraction of blood volume from the left ventricle during systole. This decreased ejection fraction results in a narrowed pulse pressure (ie, the difference between systolic and diastolic blood pressures) and weak, thready peripheral pulses. With exertion, the volume of blood that is pumped to the brain and other parts of the body is insufficient to meet metabolic demands, resulting in exertional dyspnea, anginal chest pain, and syncope. Educational objective:Aortic stenosis obstructs blood flow during systole from the left ventricle to the aorta. Clients will develop exertional dyspnea, chest pain, and syncope as the heart is unable to overcome the obstruction to pump enough blood to meet metabolic demands. A systolic ejection murmur over the aortic area, soft or absent second heart sounds, and weak peripheral pulses are characteristic.

A nurse in the intensive care unit is caring for a postoperative cardiac transplant client. What intervention is most important to include in the plan of care? 1. Apply sequential compression devices to prevent deep venous thrombosis 2. Assist client to change positions slowly to prevent hypotension 3. Encourage coughing and deep breathing to prevent pneumonia 4. Use careful hand washing and aseptic technique to prevent infection

4. Use careful hand washing and aseptic technique to prevent infection Clients receiving transplanted organs are prescribed lifelong immunosuppressive medications (eg, cyclosporine, mycophenolate) to prevent rejection. Posttransplant infection is the most common cause of death. Signs of infection may include fever >100.4 F (38 C), productive or dry cough, and changes in secretions; however, common signs of infection (eg, redness, swelling) may be absent due to immunosuppression. Critical postoperative infection control measures incorporate vigilant hand washing, aseptic technique for line/dressing changes, and possibly reverse isolation. Educational objective: Posttransplant infection is the most common cause of death due to immunosuppressive therapy. Strict hand washing and aseptic technique are critical to infection prevention. Symptoms of infection should be monitored and may include fever >100.4 F (38 C), productive or dry cough, and changes in secretions.

A client with myocardial infarction (MI) underwent successful revascularization with stent placement, is now chest pain free, and will be attending cardiac rehabilitation as an outpatient. The client is embarrassed to talk to the health care provider (HCP) about resuming sexual relations after an MI. What teaching should the nurse initiate with this client? 1. If the client is able to climb 2 flights of stairs without symptoms, the client may be ready for sexual activity if approved by the HCP 2. Inform the client that medications such as sildenafil or tadalafil are available as prescriptions from the HCP 3. It will be 6 months before the heart is healthy enough for sexual activity 4. The client will be ready for sexual activity after completion of cardiac rehabilitation

Sexual counseling is important for cardiac clients, yet can be difficult for clients and HCPs to discuss and is often neglected. Clients' concern about resumption of sexual activity can prove to be more stressful than would be the activity itself. The nurse should encourage clients to discuss concerns with the HCP; in general, if a client can walk 1 block or climb 2 flights of stairs without symptoms, the client can resume sexual activity safely. Educational objective:It is important to educate clients and their partners about sexual activity after an MI. Generally, it is safe for clients to consider resumption of sexual activity when they can walk 1 block or climb 2 flights of stairs without symptoms.

A 3-month-old client has stopped breathing. Identify the area where the nurse should check the client's pulse.

The American Heart Association's guidelines for infant cardiopulmonary resuscitation (CPR) are used on children age <1 year. To check a pulse on an infant, the nurse should palpate the brachial artery by placing 2 or 3 fingers halfway between the shoulder and elbow on the medial aspect of the arm. The pulse should be assessed for 5-10 seconds to determine its presence and quality before CPR is initiated. The brachial pulse is preferred in infants as the brachial artery is close to the surface and is easily palpable.


Conjuntos de estudio relacionados

5 - Life Insurance Underwriting and Policy Issue

View Set

Biology: QUIZ 3: BODY CONTROL AND INTERACTION WITH ENVIRONMENT

View Set

Physics Final Chapter 16 and 17 Multiple Choice

View Set

Chapter 10: Information Systems with the Organization

View Set

Test 3 (Consumer Protection) P10

View Set

Chapter 2: Chemical Level of Organization

View Set

LEADERSHIP/ MANAGEMENT practice EAQ

View Set